SlideShare a Scribd company logo
1 of 154
Download to read offline
Adhamjon’s SC notes                                                                       UZBEKISTAN




Note: These are my SC notes collected from testmagic.com. The explanations are those of 800Bob, a
Gmat instructor, who has got a perfect score. For new explanations, search on testmagic Gmat
forum. The explanations are very informative and helpful. I learned a lot from them. I thank
800Bob for such awesome explanations.
I hope my notes will help others.
All the best.




For the farmer who takes care to keep them cool, providing them with high-energy feed,
and milking them regularly, Holstein cows are producing an average of 2,275 gallons of
milk each per year.
A. providing them with high-energy feed, and milking them regularly, Holstein cows
are producing
B. providing them with high-energy feed, and milked regularly, the Holstein cow
produces
C. provided with high-energy feed, and milking them regularly, Holstein cows are
producing
D. provided with high-energy feed, and milked regularly, the Holstein cow produces
E. provided with high-energy feed, and milked regularly, Holstein cows will produce


Bob’s EXPLANATION:


B and D are easily eliminated because quot;themquot; does not agree with quot;cowquot;.


A has a number of problems:
1. The modifying phrases quot;providing them with high-energy feedquot; and quot;milking them regularlyquot; are
located between two clauses. It is not clear whether they modify the subject quot;whoquot; (= quot;farmersquot;) of the
preceding clause or the subject quot;cowsquot; of the following clause.
2. The comma between the two modifying phrases should be omitted.
3. The use of the progressive tense quot;are producingquot; is inappropriate to describe a general scientific truth.


C is wrong because quot;provided with high-energy feedquot; and quot;milking them regularlyquot; are not parallel and

                                                                                                               1
Adhamjon’s SC notes                                                                       UZBEKISTAN

because, like A, it uses the inappropriate progressive form quot;are producingquot;.


E is perfect. The three adjectives quot;coolquot;, quot;providedquot;, and quot;milkedquot; are parallel, short for: quot;to keep them
cool, keep them provided with high-energy feed, and keep them milked regularlyquot;.


And the future form quot;will producequot; is appropriate for a general scientific truth.




A mixture of poems and short fiction, Jean Toomer’s Cane has been called one of the three best
novels ever written by Black Americans—the others being Richard Wright, author of Native Son,
and Ralph Ellison, author of Invisible Man.
A. Black Americans—the others being Richard Wright, author of Native Son, and Ralph Ellison,
author of Invisible Man


B. Black Americans—including
Native Son by Richard Wright and Invisible Man by Ralph Ellison
C. a Black American—including Richard Wright, author of Native Son, and Ralph Ellison, author
of Invisible Man


D. a Black American—the others being Richard Wright, author of
Native Son, and Ralph Ellison, author of Invisible Man


E. a Black American—the others being Richard Wright’s
Native Son and Ralph Ellison’s Invisible Man




Bob’s explanation:


The best response is E.


The only potential problem with E is quot;the three best novels ever written by a Black Americanquot;. But it is
not wrong here to use the singular quot;Black Americanquot; because each of the three novels was written by just
one person. Here are a couple of perfectly correct sentences that do something similar:

                                                                                                             2
Adhamjon’s SC notes                                                                        UZBEKISTAN


According to the American Dietetic Association, children who eat a healthy breakfast are more likely to
have better concentration, problem-solving skills and eye-hand coordination.


Those with a college degree had almost twice as much income as those with just a high school diploma.


In the first example, each child eats just one breakfast. In the second, each person has just one college
degree and just one high school diploma.


Here are two correct sentences taken from Official Guides:


Three out of four automobile owners in the United States also own a bicycle. (OG10, SC 252)


Neanderthals had a vocal tract resembling an ape’s and so were probably without language, a
shortcoming that may explain why they were supplanted by our own species. (OG Verbal, SC 59)


For the first of these two examples there are a couple of answer choices that change quot;a bicyclequot; to
quot;bicyclesquot;, but as the official explanation says, quot;the plural bicycles detracts from clarity by suggesting that
multiple bicycles are owned by each person in question.quot; Similarly, for the second of these two examples
there are three wrong answer choices that change quot;a vocal tractquot; to quot;vocal tractsquot;.


So, in the example in question, choice B is wrong because it suggests that these novels each had multiple
authors. It is also wrong because quot;includingquot; is imprecise, suggesting that Native Son and Invisible Man
are Black Americans.


You cannot eliminate E just because it includes the word quot;beingquot;. It is true that quot;beingquot; is often wrong or
unnecessary, but to say that it is banned by the GMAT is ridiculous.




Climatic shifts are so gradual as to be indistinguishable at first from ordinary fluctuations in the
weather.
(A) so gradual as to be indistinguishable
(B) so gradual they can be indistinguishable

                                                                                                              3
Adhamjon’s SC notes                                                                        UZBEKISTAN

(C) so gradual that they are unable to be distinguished
(D) gradual enough not to be distinguishable
(E) gradual enough so that one cannot distinguish them




Bob’s explanation:


Use of the passive construction after able to or unable to is discouraged. It is not just awkward, but
illogical. To be distinguished from ordinary fluctuations is not an ability that climatic shifts can possess or
lack.
To take a simpler example, look at this ugly sentence:
This math problem is unable to be solved.
Better:
This math problem cannot be solved.




In no other historical sighting did Halley 's comet cause such a worldwide sensation as did its return in
1910-1911.


A did its return in 1910-1911
B had its 1910-1911 return
C in its return of 1910-1911
D its return of 1910-1911 did
E its return in 1910-1911


Bob’s explanation:
If you begin the sentence this way, you can see more easily how to finish it:
Halley's comet caused such a worldwide sensation in no other historical sighting as ...
The comparison is between other historical sightings and that of 1910-1911, so you could finish the
sentence this way:
Halley's comet caused such a worldwide sensation in no other historical sighting as it did in its return of
1910-1911.
Deleting the superfluous words quot;it didquot; you get:

                                                                                                              4
Adhamjon’s SC notes                                                                     UZBEKISTAN


Halley's comet caused such a worldwide sensation in no other historical sighting as in its return of 1910-
1911.


So it is perfectly correct to write:


In no other historical sighting did Halley's comet cause such a worldwide sensation as in its return of
1910-1911.


The correct response is C.




According to a recent study of consumer spending on prescription medications, increases in the
sales of the 50 drugs that were advertised most heavily accounts for almost half of the $20.8 billion
increase in drug spending last year, the remainder of which came from sales of the 9,850
prescription medicines that companies did not advertise or advertised
very little.


A. heavily accounts for almost half of the $20.8 billion increase in drug spending last year, the
remainder of which came
B. heavily were what accounted for almost half of the $20.8 billion increase in drug spending last
year; the remainder of the increase coming
C. heavily accounted for almost half of the $20.8 billion increase in drug spending last year, the
remainder of the increase coming
D. heavily, accounting for almost half of the $20.8 billion increase in drug spending last year, while
the remainder of the increase came
E. heavily, which accounted for almost half of the $20.8 billion increase in drug spending last year,
with the remainder of it coming


Bob’s explanation:


I wish to respectfully disagree with a couple of the points made above.


1. quot;In GMAT we look for the best and not the right answer.quot;

                                                                                                             5
Adhamjon’s SC notes                                                                      UZBEKISTAN

The idea that sometimes the 5 choices are all bad and that we are to identify the least bad choice is a myth.
There is never anything wrong with with correct response. The directions say to choose the quot;bestquot; answer,
not the quot;correctquot; answer, because there can be more than one correct way to rephrase the sentence, but
only one of the correct ways to do so will be found among the answer choices.


2. quot;B changes the meaning of the original sentence.quot;
There is nothing in the directions to Sentence Correction about finding a choice that does not change the
meaning of the original sentence. Indeed, sometimes it is necessary to change the meaning of the original,
that is, when the original is illogical. Here, for example, is the orignal sentence of OG number 95:


Unlike Schoenberg's 12-tone system that dominated the music of the postwar period, Bartók founded no
school and left behind only a handful of disciples.


In this form the sentence states that Schoenberg's 12-tone system was unlike Bartók. That's true, but it is
obviously not what the writer intended. What matters is not the literal meaning of the original sentence,
but the intended meaning. The objective is to find a choice that does not distort the intended meaning. The
challenge sometimes is to determine from a poorly written original what the writer really intended.


3. quot;...there was a run on.quot;
Choice C is not a run-on sentence. A run-on is two independent clauses treated as a single sentence. Here
what follows the last comma is:


the remainder of the increase coming from sales of the 9,850 prescription medicines that companies did
not advertise or advertised


That's not an independent clause. It could not stand alone: there's no conjugated verb. If you really want to
know what grammatical form we have here... It's a nominative absolute. Check out Nominative absolute -
Wikipedia, the free encyclopedia.


There is nothing wrong with C, and so C is the best response.


Over 75 percent of the energy produced in France derives from nuclear power, while in Germany it
is just over 33 percent.



                                                                                                              6
Adhamjon’s SC notes                                                                        UZBEKISTAN

A. while in Germany it is just over 33 percent
B. compared to Germany, which uses just over 33 percent
C. whereas nuclear power accounts for just over 33 percent of the energy produced in Germany
D. whereas just over 33 percent of the energy comes from nuclear power in Germany
E. compared with the energy from nuclear power in Germany, where it is just over 33 percent




Bob’s explanation:
A is wrong because it refers illogically to the subject of the preceding clause 75 percent of the energy
produced in France. It makes no sense to say that 75 percent of the energy produced in France is just over
33 percent in Germany.
B is wrong because it compares 75 percent of the energy produced in France with Germany.
D is wrong because it says that 33 percent of the energy produced in France comes from nuclear power in
Germany. How can energy produced in France come from Germany?
E is wrong because it is awkward and confusing, and specifically because the reference of the pronoun it
is unclear. What is just over 33 percent? If it's nuclear power that's just over 33 percent, then just over 33
percent of what?


The correct response is C




Minivans carry as many as seven passengers and, compared with most sport utility vehicles, cost
less, get better gas mileage, allow passengers to get in and out more easily, and have a smoother
ride.


A.Minivans carry as many as seven passengers and, compared with most sport utility vehicles, cost
less,
B.Minivans, which carry as many as seven passengers, compared with most sport utility vehicles,
they cost less,
C.Minivans carry as many as seven passengers, in comparison with most sport utility vehicles, and
have a lower cost, they
D.Minivans, carrying as many as seven passengers, compared with most sport utility vehicles, cost
less,



                                                                                                                 7
Adhamjon’s SC notes                                                                       UZBEKISTAN

E.Minivans, which carry as many as seven passengers, compared with most sport utility vehicles the
cost is lower, and they


Bob’s explanation:


D is awkward and confusing in that it has two modifying phrases (quot;carrying as many as seven passengersquot;
and quot;compared with most sport utility vehiclesquot;) between the subject quot;minivansquot; and the first verb quot;costquot;.


Reading the sentence as choice D phrases it, when I get to quot;comparedquot; I first think it's the verb. Only
when I get to quot;costquot; do I realize that the phrase beginning with quot;comparedquot; is not the predicate but a
second modifier. D is confusing also in that it is not clear in what way minivans are being compared with
most SUVs. Is the fact that minivans carry as many as seven passengers one of the differences?


There is absolutely nothing wrong with choice A. It tells us two things about minivans:
Minivans carry as many as seven passengers.
Minivans (compared with most SUVs) cost less, etc.
Since these two facts have the same subject, we can put them together, without repeating the subject, and
inserting quot;andquot; between the verbs:
Minivans carry as many as seven passengers and (compared with most SUVs) cost less, etc.
It would be wrong to put a comma in front of quot;andquot;. On the other hand, the comma after quot;andquot; makes
perfect sense: it's one of a pair of commas setting off the modifying phrase quot;compared with most SUVsquot;.




The fact of some fraternal twins resembling each other greatly and others looking quite dissimilar
highlights an interesting and often overlooked feature of fraternal-twin pairs, namely they vary
considerably on a spectrum of genetic relatedness.


A. The fact of some fraternal twins resembling each other greatly and others looking quite
dissimilar highlights an interesting and often overlooked feature of fraternal-twin pairs, namely
they vary considerably


B. That some fraternal twins resemble each other greatly while others look quite dissimilar
highlights an interesting and often overlooked feature of fraternal-twin pairs, namely that they vary

                                                                                                            8
Adhamjon’s SC notes                                                                        UZBEKISTAN

considerably


C. With some fraternal twins resembling each other greatly and others looking quite dissimilar, it
highlights an interesting and often overlooked feature of fraternal-twin pairs, namely considerable
variation


D. With some fraternal twins resembling each other greatly and others looking quite dissimilar, it is
a fact that highlights an interesting and often overlooked feature of fraternal-twin pairs, namely a
considerable variation


E. Because some fraternal twins resemble each other greatly and others look quite dissimilar, this
fact highlights an interesting and often overlooked feature of fraternal-twin pairs, namely they vary
considerably


EXPLANATION BY BOB:


This is a case in which the subject is a whole clause: quot;that some fraternal twins resemble each other
greatly while others look quite dissimilar.quot; That is the thing that is doing the highlighting.


A IS WRONG:
The fact of + noun + gerund...quot; - unidiomatic.
quot;The fact that + noun + verb...quot; - better.
quot;That + noun + verb....quot; - best.


Example:
quot;The fact of the GMAT being hard cannot be denied.quot; - unidiomatic and ugly.
quot;The fact that the GMAT is hard cannot be denied.quot; - OK, but wordy.
quot;That the GMAT is hard cannot be denied.quot; - best




Malaria ravages more people than any disease, yet there are only a dozen laboratories in the world
which are devoted to its study.
(A) than any disease, yet there are only a dozen laboratories in the world which
(B) than any disease, but there are only a dozen laboratories in the world which

                                                                                                        9
Adhamjon’s SC notes                                                                      UZBEKISTAN

(C) than does any disease, but only a dozen laboratories in the world
(D) as any other disease, but only a dozen laboratories in the world
(E) than any other disease, yet only a dozen laboratories in the world


EXPLANATION BY BOB:


in this case the word quot;otherquot; is enough to make it clear that the point of comparison is quot;malaria.quot; In
correct choice E:


quot;Malaria ravages more people than any other disease...quot;


there can be no ambiguity. quot;Peoplequot; are not a disease. It would not be wrong to write:


quot;Malaria ravages more people than does any other disease...quot;


but it is wrong to omit quot;other.quot;




Leaching, the recovery of copper from the drainage water of mines, as a method of the
extraction of minerals, it was well established as early as the eighteenth century, but until
about 25 years ago miners did not realize that bacteria take an active part in the process.
A. as a method of the extraction of minerals, it was well established
B. as a method of the extraction of minerals well established
C. was a well-established method of mineral extraction
D. was a well-established method of extracting mineral that was
E. had been a method of mineral extraction, well established


EXPLANATION BY BOB:


There is no way the answer to number 1 can be A. It contains a double subject: quot;Leaching... it was...quot;


In number 1, the past perfect is not necessary. The temporal relationship is made perfectly clear by the
words quot;but until about 25 years ago.quot; Another example: quot;Until the 1960s New York was the most
                                                                                                           10
Adhamjon’s SC notes                                                                        UZBEKISTAN

populous state in the Union, but in the 1970s and ’80s the state began to experience an economic decline.quot;
No need to write: quot;had been the most populous state.quot;


In any case, the past perfect makes no sense the way it is used in E in that it suggests that leaching is no
longer a method of mineral extraction. Past perfect would not be wrong, but it would have to be written
something like: quot;had been a well-established method of mineral extraction.quot;


The answer is C.




According to United States census data, while there was about one-third of mothers
with young children working outside the home in 1975, in 2000, almost two-thirds of
those mothers were employed outside the home.


A.while there was about one-third of mothers with young children working outside
the home in 1975, in 2000, almost two-thirds of those mothers were employed outside
the home.


B.there were about one-third of mothers with young children who worked outside the
home in 1975; in 2000, almost two-thirds of those mothers were employed outside the
home


in 1975 about one-third of mothers with young children worked outside the home; in
2000, almost two-thirds of such mothers were employed outside the home


even though in 1975 there were about one-third of mothers with young children who
worked outside the home, almost two-thirds of such mothers were employed outside
the home in 2000


with about one-third of mothers with young children working outside the home in
1975, almost two-thirds of such mothers were employed outside the home in 2000
                                                                                                               11
Adhamjon’s SC notes                                                                      UZBEKISTAN


OA is C.




800BOB:
a variety ofquot;
quot;a number ofquot;
quot;a total ofquot;
...plural


quot;the variety ofquot;
quot;the number ofquot;
quot;the total ofquot;
...singular


A variety of ideas were presented.
The variety of ideas was mind-boggling.




BOB:


Here are three correct sentences from 1000 SC's in which quot;thatquot; refers to a noun earlier than the
immediately preceding one:


Out of America’s fascination with all things antique has grown a market for bygone styles of furniture and
fixtures that is bringing back the chaise lounge, the overstuffed sofa, and the claw-footed bathtub.


Dr. Sayre’s lecture recounted several little-known episodes in the relations between nations that illustrate
what is wrong with alliances and treaties that do not have popular support.


In good years, the patchwork of green fields that surrounds the San Joaquin Valley town bustles with
farm workers, many of whom are in the area just for the season.




                                                                                                         12
Adhamjon’s SC notes                                                                           UZBEKISTAN




EXPLANATION BY BOB:


Relative pronoun (that, who, which) agree with the immediately preceding noun.


Subject and object pronouns (it, they, them) agree with the subject of the preceding clause or sentence


I asked someone who is far far better than I am at determining the correct rules and grammar in general
what his thoughts were on this subject and he (he's someone who had a perfect score on the exam) said,
quot;In general, a participial phrase set off from the rest of the sentence with commas is understood to modify
the subject of the sentencequot;


It is misleading and oversimplistic to say that quot;whichquot; must always be preceded by a comma. The rule is
that a nonrestrictive clause should be set off with commas, but a nonrestrictive clause does not always
begin with quot;which,quot; and the presence of quot;whichquot; does not always signal a nonrestrictive clause.


First of all, if the relative pronoun is the object of a preposition, whether the clause is restrictive or
nonrestrictive, you cannot use quot;thatquot;; you must use quot;which.quot; Here are a couple of correct sentences from
the OG that use quot;whichquot; after a preposition in a restrictive clause:


El Nino, the periodic abnormal warming of the sea surface off Peru, is a phenomenon in which changes in
the ocean and atmosphere combine to allow the warm water that has accumulated in the western Pacific
to flow back to the east.


As U.S. nuclear attack submarines prowl their familiar haunts deep within the oceans of the world these
days, they increasingly are engaged in missions far different from the tasks for which they were built and
for which their crews were trained over the last forty years




Chicago, where industrial growth in the nineteenth century was more rapid than any other
American city, was plauged by labour troubles like the Pullman Strikes of 1894.
A) where industrial growth in the nineteenth century was more rapid than any other American city

                                                                                                             13
Adhamjon’s SC notes                                                                       UZBEKISTAN

B) which had industrial growth in the nineteenth century more rapid than that of other American
cities
C) which had growth industurally more rapid than any other American city in the ninteenth
century
D) whose industrial growth in the nineteenth century was more rapid than any other American city
E) whose industrial growth in the nineteenth century was more rapid than of any
other American city


EXPLANATION BY BOB:


Something's wrong here. All choices are wrong. Most other choices have comparison problems.


A: says that Chicago's growth was quot;more rapid than any other American city.quot; Needs to be quot;more rapid
than that of any other American city.quot;


B: quot;had industrial growth... more rapid than that of other American cities.quot; Here quot;that ofquot; makes no sense.
If you're going to start the comparison with quot;had,quot; you need to finish with a verb, as in quot;than other
American cities had.quot;


C: Adverb quot;industriallyquot; in quot;had growth industrially more rapidquot; seems to modify quot;more rapidquot; instead of
quot;had growth.quot;


D: like A, needs quot;that of.quot;


E: Maybe this is supposed to be the correct answer and it's just mistyped. I see quot;ofquot; where I want to see
quot;that of.quot;


You can say either:
which had industrial growth more rapid than other American cities had
or:
whose industrial growth was more rapid than that of other American cities
But B confuses the two. quot;That of other American citiesquot; needs to be compared with some sort of
possessive form, such as quot;the industrial growth of whichquot; or quot;whose industrial growth.quot;



                                                                                                            14
Adhamjon’s SC notes                                                                 UZBEKISTAN

Maybe an analogous but simpler example will help. You can say either:
The area of Russia is greater than that of India.
or:
Russia has a greater area than India has.
But it's imprecise to say:
Russia has a greater area than that of India.




BOB:


before Australia was Australia, it was the antipodes MEANS
It means: before the country that is now know as Australia got that name.


This is not such a strange expression. Try googling on quot;before America was Americaquot; or quot;before England
was Englandquot; and you'll get hundreds of hits.




Although all the proceedings of the Communist party conference held in Moscow were not carried


live, Soviet audiences have seen a great deal of coverage.


(A) all the proceedings of the Communist party conference held in Moscow were not carried live


(B) all the Communist party conference's Moscow proceedings were not carried live


(C) all the Communist party conference Moscow proceedings have not been carried alive


(D) not all the Communist party conference Moscow proceedings have been carried alive


(E) not all the proceedings of the Communist party conference held in Moscow were carried live


What is wrong with th given sentence? The OA is E



                                                                                                    15
Adhamjon’s SC notes                                                                      UZBEKISTAN


EXPLANATION BY BOB:


A and E do not mean the same thing.


A, which says that quot;all the proceedings were not carried live,quot; means that no proceeding was carried live.
Though not grammatically incorrect, A is awkward and clearly not what the writer intended, since
quot;audiences have seen a great deal of coverage.quot;


There is a difference between quot;All X are not Yquot; and quot;Not all X are Y.quot; Consider the following:


1. All of us are not going.
2. Not all of us are going.


In the first example, no one is going. In the second, some, but not all, are going.


The construction used in A is a mistake commonly made by native speakers. Here are a few examples I
found on the Internet:


1. quot;Althen is quick to acknowledge that all Americans are not white and middle-class.quot;
Wrong. Should be: quot;...not all Americans are white and middle-class,quot; or quot;...Americans are not all white
and middle-class.quot;


2. quot;All prime numbers are not odd.quot;
Wrong. Should be: quot;Not all prime numbers are oddquot;, or quot;Prime numbers are not all odd.quot;


3. quot;All languages are not based on the English alphabet.quot;
Wrong. Should be: quot;Not all languages are based on the English alphabet,quot; or quot;Languages are not all based
on the English alphabet.quot;




In a period of time when women typically have had a narrow range of choices, Mary Baker Eddy
became a distinguished writer and the founder, architect, and builder of a growing church.

                                                                                                          16
Adhamjon’s SC notes                                                                      UZBEKISTAN


(A) In a period of time when women typically have
(B) During a time in which typically women have
(C) Typically, during a time when women
(D) At a time when women typically
(E) Typically in a time in which women




EXPLANATION BY BOB:


and B are wordy, but the real reason to eliminate them is quot;have.quot; The present perfect is wrong when
describing an event that happened and was completed in the past. C and E are wrong because quot;typicallyquot;
is misplaced.


Quote:

Originally Posted by zombie_in
As per rules whenever we use quot;whichquot; it should have comma(,) in front of it


Not true. You don't put a comma in front of quot;whichquot; when it follows a preposition. quot;In whichquot; is perfectly
fine.


California remains the state in which most Americans wish they could live.
A historical novel is a novel in which the story is set among historical events.


Would you really put a comma in front of which?


California remains the state in, which most Americans wish they could live. (sic)
A historical novel is a novel in, which the story is set among historical events. (sic




Believed to originate from a small area on their foreheads, elephants emit low-frequency sounds
that may be used as a secret language to communicate with other members of the herd.
(A) Believed to originate from a small area on their foreheads, elephants emit low-frequency sounds
that may be used

                                                                                                        17
Adhamjon’s SC notes                                                                      UZBEKISTAN

(B) Elephants emit low-frequency sounds that are believed to originate from a small area on their
foreheads, and they may use this
(C) Elephants emit low-frequency sounds, believed to originate from a small area on their
foreheads, that they may use
(D) Originating, it is believed, from a small area on their foreheads; elephants emit low-frequency
sounds they may use
(E) Originating, it is believed, from a small area on their foreheads, low-frequency sounds are
emitted by elephants that may be used


isnt 'they' in choice B refers to subject 'elephant' ........




EXPLANATION BY BOB:


Problem with B is quot;this,quot; which is supposed to refer to quot;sounds.quot;


Best response is C.




Sunspots, vortices of gas associated with strong electro-magnetic activity, are visible as dark spots
on the surface of the Sun but have never been sighted on the Sun's poles or equator.
(A) are visible as dark spots on the surface of the Sun but have never been sighted on
(B) are visible as dark spots that never have been sighted on the surface of the Sun
(C) appear on the surface of the Sun as dark spots although never sighted at
(D) appear as dark spots on the surface of the Sun, although never having been sighted at
(E) appear as dark spots on the Suns surface, which have never been sighted on


EXPLANATION:


If you want quot;although never sightedquot; to modify quot;sunspots,quot; you would need to move it to the beginning of
the sentence. Placed at the end, quot;althoughquot; needs to be followed by a subject and conjugated verb. quot;Atquot;
versus quot;onquot; is a red herring -- they're both OK.


It is counterproductive to disagree with the OG. The people who write the test say the answer is A. It's not

                                                                                                          18
Adhamjon’s SC notes                                                                       UZBEKISTAN

a typographical error. They offer a detailed explanation for why A is right and C is wrong. If one wants to
get a high score on the GMAT, one needs to learn to think the way the test makers think.




In the most common procedure for harvesting forage crops such as alfalfa, as much as 20 percent of
the leaf and small-stem material, which is the most nutritious of all the parts of the plant, shattered
and fell to the ground.
(A) which is the most nutritious of all the parts of the plant, shattered and fell
(B) the most nutritious of all parts of the plant, shatter and fall
(C) the parts of the plant which were most nutritious, will shatter and fall
(D) the most nutritious parts of the plant, shatters and falls
(E) parts of the plant which are the most nutritious, have shattered and fallen
OA D


EXPLANATION BY BOB:


There are two ways to look at the subject quot;20 percent of the leaf and small-stem material.quot;


One way is to think it means 20 percent of two different things: quot;leafquot; and quot;small-stem material.quot; This
way doesn't work because you can't say quot;20 percent of the leaf.quot; quot;Leafquot; is a countable noun. It would have
to be quot;20 percent of the leaves.quot;


The other way is to think it means 20 percent of the material. The material consists of leaves and small
stems, and so it can be called quot;leaf and small-stem material.quot; This way works.


So for me, the subject is singular. The present tense also works for me. So I would choose D. But I have a
problem with D. If quot;leaf and small-stem materialquot; is singular, then I think it would be better for the
following appositive to be singular also: quot;the most nutritious part of the plant.quot;


All in all, not a good question. Clearly not GMAT-quality.




                                                                                                           19
Adhamjon’s SC notes                                                                      UZBEKISTAN

                           Pronoun 'they'


                           1. While depressed property values can hurt some large investors, they are
                           potentially devastating for home-owners, whose equity—in many cases
                           representing a life’s savings—can plunge or even disappear.


                           (A) they are potentially devastating for homeowners, whose


                           (B) they can potentially devastate homeowners in that their


                           (C) for homeowners they are potentially devastating, because their


                           (D) for homeowners, it is potentially devastating in that their(A)


                           (E) it can potentially devastate homeowners, whose


                           Why is (A) correct? the use of 'they' can either represent 'proporty values' or
                           'large investors'. Please let me know.




EXPLANATION BY BOB:




Plural pronouns do not necessarily refer to the nearest plural noun, and singular pronouns do not
necessarily refer to the nearest singular noun. If that were true, there would be no such thing as ambiguous
pronoun reference. In the OG alone there are dozens of SC items in which the correct answer choice
includes a pronoun whose antecedent is not the nearest available noun. In addition to the item quoted
above (SC 159), here are just six more:


97. Iguanas have been an important food source in Latin America since prehistoric times, and they are
still prized as game animals by the campesinos....
The pronoun “they” refers to “iguanas,” not to the nearest plural noun “times.”



                                                                                                             20
Adhamjon’s SC notes                                                                         UZBEKISTAN

103. Students in the metropolitan school district are so lacking in math skills that it will be difficult to
absorb them into a city economy....
The pronoun “them” refers to “students,” not to the nearest plural noun “skills.”


152. ...a microbe never before seen on Earth that might escape from the laboratory and kill vast numbers
of humans who would have no natural defenses against it.
The pronoun “it” refers to “microbe,” not to the nearest singular noun “laboratory.”


153. A recording system was so secretly installed and operated in the Kennedy Oval Office that even
Theodore C. Sorensen, the White House counsel, did not know it existed.
The pronoun “it” refers to “system,” not to the nearest singular noun “Office.”


180. Quasars are so distant that their light has taken billions of years to reach the Earth; consequently,
we see them as they were during the formation of the universe.
The pronouns “them” and “they” refer to “quasars,” not to the nearest plural noun “years.”


251. The gyrfalcon, an Arctic bird of prey, has survived a close brush with extinction; its numbers are now
five times greater than when....
The pronoun “its” refers to “gyrfalcon,” not to the nearest singular noun “extinction.”


The rule is not so simply stated as quot;a pronoun will refer to the nearest available noun.quot; In fact, as all the
above examples show, if a pronoun agrees in number with the subject of the preceding clause, then that
subject will normally be the antecedent, even if there is another noun of the same number closer to the
pronoun.




According to GMAT which is correct
This is me. or This is I.


EXPLANATION BY BOB:


I can't recall ever seeing either the word quot;mequot; or the word quot;Iquot; in a GMAT Sentence Correction item.



                                                                                                                21
Adhamjon’s SC notes                                                                       UZBEKISTAN

Strictly speaking, you should use the subject pronoun after forms of the verb quot;be.quot;


It is I.
It was they who arrived first.
quot;Hello. May I speak to Bob?quot; quot;This is he.quot;
If you were I...


This is because quot;bequot; is not a transitive verb. A pronoun after quot;bequot; is a complement, not a direct or indirect
object, and so grammatically speaking it is incorrect to use an object pronoun.


But only grammar pedants like me speak this way. Normal people say, quot;It's me!quot;


In any case, don't worry. This is never tested on the GMAT.




In large doses, analgesics that work in the brain as antagonists to certain chemicals have caused
psychological disturbances in patients, which may limit their potential to relieve severe pain.
(A) which may limit their potential to relieve
(B) which may limit their potential for relieving
(C) which may limit such analgesics’ potential to relieve
(D) an effect that may limit their potential to relieve
(E) an effect that may limit the potential of such analgesics for relieving


I have a question on this problem, I was able to get till D & E and then picked D thinking that 'their' refers
to analgesics because as told by Bob, pronoun refers to subject of previous clause, which is quot;analgesicsquot; .
Also E looks to me lengthy
plz advise where i'm going wrong...thanx!!




EXPLANATION BY BOB:


I say E.



                                                                                                            22
Adhamjon’s SC notes                                                                          UZBEKISTAN

This is an interesting item. I'm not sure it's a real GMAT item, but I still think it's a useful one for
illustrating the dangers of following a rule of grammar too literally. Those of us with extremely analytical
minds would like to think that the rules of grammar can be expressed with precision. We must face the
fact, however, that language is not strictly logical.


I stand by the guideline that a pronoun will tend to refer to the subject of the preceding phrase if the
pronoun and subject agree in number. (We have seen many, many examples where the pronoun does not
automatically refer to the nearest noun.) So in this case it would seem that quot;theirquot; in choice D refers
grammatically and logically to quot;analgesics.quot; The problem with D, however, is that there are just too many
other plural nouns between quot;analgesicsquot; and quot;theirquot;: quot;antagonists,quot; quot;chemicals,quot; quot;disturbances,quot;
quot;patients.quot; For this reason it is better to write quot;of such analgesicsquot; than quot;their.quot;


Here's what I tell my students: If you have a choice that uses a pronoun and a choice repeats the noun,
generally you should go for the one that repeats the noun. A little repetition for the sake of clarity is not a
bad thing.




Classical guitar was neither prestigious nor was often played in concert halls until it was revived by
Andres Segovia in the mid-twentieth century, having been won over by the instrument's sound
despite its relative obscurity.


A. Classical guitar was neither prestigious nor was often played in concert halls until it was revived
by Andres Segovia in the mid-twentieth century, having been won over by the instrument's sound
despite its relative obscurity.
B. Classical guitar was neither prestigious nor played often in concert halls until it was revived by
Andres Segovia in the mid-twentieth century, having been won over by the instrument's sound
despite its relative obscurity.
C. Classical guitar was not prestigious and was not often played in concert halls until Andres
Segovia revived it in the mid-twentieth century, after he was won over by the sound despite the
instrument's relative obscurity.
D. Classical guitar did not have prestige nor was it performed often in concert halls until its revival
by Andres Segovia, who in the mid-twentieth century was won over by the instrument's sound
despite its relative obscurity.

                                                                                                              23
Adhamjon’s SC notes                                                                     UZBEKISTAN

E. Classical guitar was neither prestigious nor was often played in concert halls until Andres
Segovia revived it in the mid-twentieth century, when he was won over by the sound of the relatively
obscure instrument.
OA C


EXPLANATION BY BOB:


For me the best choice is C.


A and E are wrong because quot;neither prestigious nor was often playedquot; is not parallel.


A and B are wrong because the participial phrase quot;having been won...quot; seems to modify (illogically) the
subject of the sentence, quot;classical guitar.quot;


D is wrong because quot;playedquot; is changed to quot;performed.quot; You don't perform an instrument; you play it or
perform on it. D also has an ambiguous quot;its.quot;


I see nothing wrong with C.




With its plan to develop seven and a half acres of shore land, Cleveland is but one of a large number
of communities on the Great Lakes that is looking to its waterfront as a way to improve the quality
of urban life and attract new businesses.


(A) is looking to its waterfront as a way to improve the quality of urban life and attract


(B) is looking at its waterfront to improve the quality of urban life and attract


(C) are looking to their waterfronts to improve the quality of urban life and attract


(D) are looking to its waterfront as a way of improving the quality of urban life and attracting




                                                                                                          24
Adhamjon’s SC notes                                                                     UZBEKISTAN

(E) are looking at their waterfronts as a way they can improve the quality of urban life and attract


OA C


EXPLANATION BY BOB:


The verb must be quot;arequot;: Cleveland is one of many communities that are... quot;Thatquot; refers to quot;communitiesquot;
and is therefore plural.


Note the difference between quot;one of the [plural noun] isquot; and quot;one of the [plural noun] that arequot;:


One of the most powerful driving forces behind recycling is the threat of legislation that would require
companies to take more responsibility for the disposal of their products.


Twenty-two feet long and 10 feet in diametier, the AM-1 is one of the many new satellites that are part of
a 15-year effort to subject the interactions of Earth's atmosphere, oceans, and land surfaces to detailed
scrutiny from space.


Last edited by 800Bob : 07-08-2005 at 06:20 PM.
Looking at: Physically act of seeing sthg/someone
Looking to: Depending on, turning to, expecting sthg from..
Look to the waterfront to improve my business.. I have started considering that waterways are a good
means to improve business..
Look up to you - Turn to you for advice..




Are they both acceptable?


Mr. King agrees it is possible that planets orbiting .... could ...


Mr. King agrees that it is possible for planets orbiting ... to ....


Any ideas?



                                                                                                            25
Adhamjon’s SC notes                                                                        UZBEKISTAN


EXPLANATION BY BOB:


quot;It is possible that planets couldquot; is redundant. It's better not to use quot;could,quot; quot;might,quot; or quot;mayquot; with
quot;possiblequot; or quot;possibly.quot;


Bob, is it okay to say


It is possible for him to do something


I am not sure about this ''for x to do'' usage. What do u say?




Yes, it is possible for you to say that.




A 1972 agreement between Canada and the United States reduced the amount of phosphates that
municipalities had been allowed to dump into the Great Lakes.


(A) reduced the amount of phosphates that municipalities had been allowed to dump
(B) reduced the phosphate amount that municipalities had been dumping
(C) reduces the phosphate amount municipalities have been allowed to dump
(D) reduced the amount of phosphates that municipalities are allowed to dump
(E) reduces the amount of phosphates allowed for dumping by municipalities


EXPLANATION BY BOB:


agree with D. An agreement cannot reduce the amount that had been allowed. That previous amount
cannot be changed. There is an allowable amount. That amount was bigger before the agreement, and it is
smaller now as a result of the agreement.


Analogy: Let's say the legislature has changed the speed limit. Which sounds better?



                                                                                                           26
Adhamjon’s SC notes                                                                       UZBEKISTAN

1) The legislature increased the maximum speed that one had been allowed to drive.
2) The legislature increased the maximum speed that one is allowed to drive.


I'll go with 2.
Past perfect is used to suggest that something is over and done with before the main action of the past. It
would be OK to say:


The 1972 agreement replaced the amount of phosphates that had been allowed with a new amount.


But if you are talking about changing an amount, then the amount existed before and continues to exist
after the 1972 agreement. The present tense is used to refer to something that exists for all time.


Copernicus revealed that the Earth and the planets all revolve around the Sun.


quot;Revolvequot; is in the present tense because it is an action that was then and continues to take place. It would
be wrong to say: quot;...revealed that the Earth and the planets revolved...quot; or quot;had revolved.quot;


In the sentence under dicussion, there was and continues to be an amount of phosphates that municipalities
can dump. The 1972 agreement reduced the amount, but the amount continues to exist.


I'll try one more example. Suppose I started a new diet last week. I formerly ate all the red meat I wanted.
Under the new diet I allow myself to eat only 100 grams a day.


The diet I started last week reduced the amount of red meat I am allowed to eat to 100 grams a day.


It would be wrong to say:


The diet I started last week reduced the amount of red meat I had been allowed to eat to 100 grams a day.


GOOD INSIGHT
How can a agreement reduce something that has already been dumped?
Choice D is perfect.
A is ,initially, awfully tempting.
Brilliant Question!!


                                                                                                              27
Adhamjon’s SC notes                                                                     UZBEKISTAN




17. Bob Wilber became Sidney Bechet’s student and protege when he was nineteen and, for a few
years in the 1940’s, came as close to being a carbon copy of the jazz virtuoso in performance as
anyone has ever come.


(A) as anyone has ever come


(B) as anyone ever had been


(C) as anyone ever had done


(D) that anyone ever did


(E) that anyone ever


OA-C


EXPLANATION BY BOB:


The quot;OAquot; is wrong. The best answer is A.


B and C are both wrong because quot;everquot; is misplaced. Adverbs of frequency (quot;always,quot; quot;never,quot; quot;often,quot;
quot;seldom,quot; quot;ever,quot; etc.) must be placed between the auxiliary verb and the past participle. Should be: quot;as
anyone had ever done.quot;


There is nothing wrong with using the present perfect quot;has comequot; in A. It means that Wilber came as close
to being a carbon copy of the jazz virtuoso as anyone had come before or has come since.


Consider the following examples:


quot;In 1960 Johnson scored as many goals as anyone had ever scored in one year.quot;
quot;In 1960 Johnson scored as many goals as anyone has ever scored in one year.quot;



                                                                                                            28
Adhamjon’s SC notes                                                                     UZBEKISTAN

In the first example, Johnson's number of goals in 1960 is compared only with previous years. In the
second example his number of goals in 1960 is compared with all years before and since.
Either past perfect or present perfect would be correct. But C puts quot;everquot; in the wrong place and so cannot
be correct.


I see nothing wrong with A. For a few years in the 1940s Wilber came very close to being a carbon copy
of his teacher. To this day, no one has ever come closer.


I have seen quite a few cases in which the so-called quot;OAquot; is wrong.




1. A survey by the National Council of Churches showed that in 1986 there were 20,376 female
ministers, almost 9 percent of the nation's clergy, twice as much as 1977.
(A) twice as much as 1977
(B) twice asn many as 1977
(C) double what it was in 1977
(D) double the figure for 1977
(E) a number double that of 1977's

SPOILER: OA - D


2. According to surveys by the National Institute on Drug Abuse, about 20 percent of young adults
used cocaine in 1979, doubling those reported in the 1977 survey.


(A) doubling those reported in the 1977 survey


(B) to double the number the 1977 survey reported


(C) twice those the 1977 survey reported


(D) twice as much as those reported in the 1977 survey


(E) twice the number reported in the 1977 survey


                                                                                                         29
Adhamjon’s SC notes                                                                          UZBEKISTAN

SPOILER: OA - E


EXPLANATION BY BOB:


Yes, the dictionary says that quot;twicequot; is an adverb, but it's perfectly idiomatic to say quot;twice the number.quot;
Putting quot;twicequot; before quot;the numberquot; does not make quot;twicequot; an adjective. If it were an adjective you'd have
to place it after quot;the,quot; but you can't say quot;the twice number.quot;




VERY GOOD POINT HERE:
In April 1997, Hillary Rodham Clinton hosted an all-day White House scientific conference on new
findings that indicates a child’s acquiring language, thinking, and emotional skills as an active
process that may be largely completed before age three.



   A. that indicates a child’s acquiring language, thinking, and emotional skills as

   B. that are indicative of a child acquiring language, thinking, and emotional skills as

   C. to indicate that when a child acquires language, thinking, and emotional skills, that it is

   D. indicating that a child’s acquisition of language, thinking, and emotional skills is

   E. indicative of a child’s acquisition of language, thinking, and emotional skills as




GOOD EXPLANATION BY BOB:


I don't like E at all. quot;Indicative ofquot; is idiomatically correct in some cases, but not here. quot;Indicative ofquot;
should be followed by a noun.


Example: Some believe that facial characteristics are indicative of personality traits.


The noun quot;personality traitsquot; is what is indicated.


Example: Are these averages indicative of what a typical family pays for medical care?


The noun phrase quot;what a typical family paysquot; is what is indicated.

                                                                                                               30
Adhamjon’s SC notes                                                                              UZBEKISTAN


But to say: quot;indicative of a child's acquisition of ... skills as an active processquot; is awkward and imprecise.
It's not the skills that are indicated; it's the fact that these skills are an active process.


I see nothing wrong with D. The objection some have raised is that the participial phrase quot;indicating that a
child's acquisition...quot; might could be felt to modify the subject quot;Hillary Rodham Clintonquot; instead of the
immediately preceding noun quot;findings.quot; For a participial phrase at the end of a sentence to modify a
distant subject instead of an adjacent noun, it must be separated from the rest of the sentence with a
comma, like this:


Hillary Rodham Clinton hosted an all-day White House scientific conference on new findings, indicating
that a child’s acquisition of language, thinking, and emotional skills is an active process that may be
largely completed before age three.


Without a comma between quot;findingsquot; and quot;indicating,quot; choice D is not ambiguous




The health commissioner said that the government had implemented strict measures to eradicate
the contaminated food and, despite the recent illnesses, it will try to prevent the outbreak from
reccurring in the future.


A) it will try


B) that it tried


C) it had tried


D) it would try


E) that it would try


OA-E


EXPLANATION :

                                                                                                            31
Adhamjon’s SC notes                                                                        UZBEKISTAN


quot;Wouldquot; is used to refer to the future from the point of view of the past. quot;He said he would arrive shortly.quot;


The Health commissioner said X and Y ...


X == that the government had implemented strict measures to eradicate the contaminated food


Y= that the government would try ..


I hope this will make u clear smudge ...


A good Question . Testing Parallelism .. I was also wrong initiallly , Now i learned it.


Thanks for the question.




Employment costs rose 2.8% in the 12months that ended in sept,slightly(less than they did) in the
year that ended in the previous quarter.
A......
B.lower than
C.lower than they were
D.lower than they did
E.less than


OA-A


EXPLANATION BY BOB:


The sentence is not comparing employment costs of two different periods. It is comparing how much the
costs rose in two different periods. Employment costs rose by a smaller amount in the 12 months that
ended in September than they rose in the 12 months that ended in June. Employment costs rose 2.8% in
the more recent period, slightly less than they rose in the earlier period.




                                                                                                          32
Adhamjon’s SC notes                                                                  UZBEKISTAN




There seem to be two different versions of this item in GMATPrep. One is the version imperfectly typed
at the top of this thread. That version actually looks like this:


In addition to her work on the Miocene hominid fossil, Mary Leakey contributed to archaeology
through her discovery of the earliest direct evidence of hominid activity and through her
painstaking documentation of East African cave paintings.


A) Mary Leakey contributed to archaeology through her discovery of the earliest direct evidence of
hominid activity and through her painstaking documentation of


B) Mary Leakey contributed to archaeology by her discovery of the earliest direct evidence of
hominid activity and painstakingly documenting


C) Mary Leakey was a contributor to archaeology by discovering the earliest direct evidence of
hominid activity and with her painstaking documentation of


D) Mary Leakey's contributions to archaeology include her discovery of the earliest direct evidence
of hominid activity and painstakingly documenting


E) Mary Leakey's contributions to archaeology include her discovering the earliest direct evidence
of hominid activity and painstaking documentation of


Correct response:
SPOILER: A


The other version looks like this:


In addition to her work on the Miocene hominid fossil record, Mary Leakey contributed to
archaeology with her discovery of the earliest direct evidence of hominid activity and painstakingly
documenting East African cave paintings.


A) Leakey contributed to archaeology with her discovery of the earliest direct evidence of hominid

                                                                                                     33
Adhamjon’s SC notes                                                                     UZBEKISTAN

activity and painstakingly documenting


B) Leakey contributed to archaeology by her discovery of the earliest direct evidence of hominid
activity and by painstakingly documenting


C) Leakey was a contributor to archaeology with her discovery of the earliest direct evidence of
hominid activvity and with her painstaking documentation of


D) Leakey's contributions to archaeology include her discovery of the earliest direct evidence of
hominid activity and her painstaking documentation of


E) Leakey's contributions to archaeology include discovering the earliest direct evidence of hominid
activity and painstaking documentation of


Correct response:
SPOILER: D




Dr. Sayre’s lecture recounted several little-known episodes in the relations between nations that
illustrates what is wrong with alliances and treaties that do not have popular support.
(A) relations between nations that illustrates
(B) relation of one nation with another that illustrates
(C) relations between nations that illustrate
(D) relation of one nation with another and illustrate
(E) relations of nations that illustrates


EXPLANATION:
The correct response is C.


Subject-verb agreement problem in A, B, and E. Should be: quot;...several little-known episodes ... that
illustrate...quot;


quot;Betweenquot; is just fine here. From The Columbia Guide to Standard American English:



                                                                                                       34
Adhamjon’s SC notes                                                                       UZBEKISTAN

quot;It is often argued that between should be used to express a relationship involving two of something, and
among should express relationships involving three or more, but in fact that generalization does not
describe the way English has long used these prepositions. Between can be used of as many items as you
like if the relationship is one-to-one, however much it may be repeated with different partners: Economic
relations between Great Britain, France, and Italy [or between some members of the EEC] are tense at
present.quot;




The computer software being designed for a project studying native American access to higher
education not only meets the needs of that study, but also has the versatility and power of
facilitiating similar research endeavors.


a. but also has the versatility and power of facilititating
b. but also have the power to facilitate
c. but it also has the versatility and power to facilitate
d. and also have the versatility and power of facilititating
e. it also has such versatility and power that it can facilitate


The answer is
SPOILER: B
but I think it should be
SPOILER: C


EXPLANATION BY BOB:


The correctly typed sentence reads like this:


The computer software being designed for a project studying Native American access to higher education
will not only meet the needs of that study, but also have the versatility and power to facilitate similar
research endeavors.




It doesn't matter whether the subject is singular or plural. After will you must write have, never has.



                                                                                                            35
Adhamjon’s SC notes                                                                          UZBEKISTAN

Q; C is still correct then, not just not idiomatically?


No. The question as typed at the top of this thread has no correct answer. quot;Not only meetsquot; and quot;but it also
hasquot; are not parallel. It should be quot;not only meetsquot; and quot;but also has.quot;


A is close, but quot;power of facilitatingquot; is not idiomatic. Should be quot;power to facilitate.quot;




Not only did the systematic clearing of forests in the United States create farmland (especially in the
Northeast) and gave consumers relatively inexpensive houses and furniture, but it also caused
erosion and very quickly deforested whole regions.


1. Not only did the systematic clearing of forests in the United States create farmland (especially in
the Northeast) and gave consumers relatively inexpensive houses and furniture, but it also
2. Not only did the systematic clearing of forests in the United States create farmland (especially in
the Northeast), which gave consumers relatively inexpensive houses and furniture, but also
3. The systematic clearing of forests in the United States, creating farmland (especially in the
Northeast) and giving consumers relatively inexpensive houses and furniture, but also
4. The systematic clearing of forests in the United States created farmland (especially in the
Northeast) and gave consumers relatively inexpensive houses and furniture, but it also
5. The systematic clearing of forests in the United States not only created farmland (especially in the
Northeast), giving consumers relatively inexpensive houses and furniture, but it


EXPLANATION BY BOB:


B is no good. quot;Not only did the systematic clearing... create...quot; doesn't parallel quot;but also gave...quot;
Furthermore, B erroneously uses quot;whichquot; to refer to the whole preceding clause.


The correct response is D.


ANOTHER EXPLANATION BY BOB:


I think the best choice is D. The first clause says that the systematic clearing did some good things
(created farmland and provided inexpensive houses and furniture); the second clause says that it also did

                                                                                                         36
Adhamjon’s SC notes                                                                        UZBEKISTAN

some bad things (caused erosion and deforested whole regions). quot;Not alsoquot; generally calls for a later quot;but
also,quot; but quot;but alsoquot; does not necessarily call for an earlier quot;not only.quot; Example:


Rio de Janeiro has some of the world's most famous beaches, but it also has some of the world's most
notorious slums




New hardy varieties of rice show promise of producing high yields without the costly requirements
of irrigation and application of commercial fertilizer by earlier high-yielding varieties.


(A) requirements of irrigation and application of commercial fertilizer by earlier high-yielding
varieties


(B) requirements by earlier high-yielding varieties of application of commercial fertilizer and
irrigation


(C) requirements for application of commercial fertilizer and irrigation of earlier high-yielding
varieties


(D) application of commercial fertilizer and irrigation that was required by earlier high-yielding
varieties


(E) irrigation and application of commercial fertilizer that were required by earlier high-yielding
varieties


EXPLANATION BYBOB:


The word order in C is really bad. quot;Costlyquot; is placed before and therefore modifies quot;requirementsquot; instead
of quot;irrigation and application of commercial fertilizer.quot; Does quot;application of commercial fertilizer and
irrigationquot; mean quot;(1) application of commercial fertilizer and (2) irrigationquot; or does it mean quot;application
of (1) commercial fertilizer and (2) irrigationquot;? And then there's the placement of the phrase quot;of earlier
high-yielding varieties.quot; Does only quot;irrigationquot; relate to the earlier varieties, or do both quot;applicationquot; and
quot;irrigationquot; relate to them?



                                                                                                             37
Adhamjon’s SC notes                                                                        UZBEKISTAN

What's wrong with E? Passive? So what? It's a persistent myth that passive is bad. Sometimes passive is
not only acceptable, but necessary. Look for example at OG11 SC questions 67 and 86




By the time peace and happiness will have come to the planet,many lives will be wasted.


A......................
B. come to the planet, many lives will have been wasted
C. will have come to the planet, many lives will have been wasted
D. shall have come to the planet, many lives shall be wasted
E. would have come to the planet, many lives would have been wasted


EXPLANATION BY BOB:


I agree with the OA.


In the first clause we need the present tense. With quot;by the time,quot; as with quot;when,quot; quot;before,quot; quot;while,quot;
quot;after,quot; and quot;until,quot; we use the present with a future meeting.


By the time we get there, the store will be closed.
When I see you tomorrow, I will give you the dollar I owe you.
He will turn off the lights before he leaves.
Will you take care of the plants while I am on vacation?
We won't go until we are finished.


So already we see that the correct response must be B.


In the second clause the future perfect quot;will have beenquot; is correct. The future perfect refers to an event that
happens in the past from the point of view of the future. The time when peace and happiness come is in
the future. Lives will be wasted before that time.




                                                                                                            38
Adhamjon’s SC notes                                                                    UZBEKISTAN


One pervasive theory explains the introduction of breakfast cereals in the early 1900s as a result of
the growing number of automobiles, which led to a decline in horse ownership and a subsequent
grain glut; by persuading people to eat what had previously been horse feed, market equilibrium
was restored.


(A) by persuading people to eat what had previously been horse feed, market equilibrium was
restored


(B) persuading people to eat what had previously been horse feed restored market equilibrium


(C) by persuading people to eat what had previously been horse feed, it restored market equilibrium


(D) the persuasion of people to eat what had previously been horse feed restored market
equilibrium


(E) market equilibrium was restored when people were persuaded to eat former horse feed


EXPLANATION BY BOB:


The correct response is indeed B.


A is wrong because the phrase quot;by persuading...quot; illogically modifies quot;market equilibrium.quot;


C is wrong because quot;itquot; has no logical antecedent.


D is wrong because quot;persuasion of peoplequot; is ambiguous. Are the people persuading or are they being
persuaded?


E is wrong for a couple of reasons:
1) The word quot;whenquot; suggests that quot;market equilibrium was restoredquot; and quot;people were persuadedquot; merely
happened concurrently. The writer's intent is to say that quot;people were persuadedquot; was the cause of quot;market
equilibrium was restored.quot;
2) quot;Former horse feedquot; is awkward and imprecise. Sounds like feed for former horses.


                                                                                                       39
Adhamjon’s SC notes                                                                            UZBEKISTAN




As the honeybee's stinger is heavily barbed, staying where it is inserted, this results in the act of
stinging causing the bee to sustain a fatal injury.


A.As the honeybee's stinger is heavily barbed, staying where it is inserted, this results in the act of
stinging causing


B.As the heavily barbed singer of the honey bee stays where it is inserted, with the result that the act
of stinging causes


C.The honeybee's stinger, heavily barbed and staying where it is inserted, results in the fact that the
act of stinging causes


D.The heavily barbed stinger of the honeybee stays where it is inserted, and results in the act of
stinging causing


E.The honeybee's stinger is heavily barbed and stays where it is inserted, with the result that the act
of stinging causes


EXPLANATION BY BOB:


I do not agree with (C). It is not the bee's stinger that causes the bee's death. It is the fact that the stinger is
barbed and stays where it is inserted. For me, the best answer is (E).


Seems like a quot;GMAT qualityquot; question to me. It has four absolutely wrong answer choices and one
answer choice with absolutely nothing wrong.


(A) is wrong because it uses quot;thisquot; to refer not to a specific noun but to the whole idea expressed earlier in
the sentence. (A) is also wrong because of quot;results in the act of stinging causing...quot; quot;Causingquot; is a gerund
and needs to be preceded by the possessive, which would be the very ugly quot;the act of stinging's causing...quot;
The only way around that would be to write something like quot;results in the fact that the act of stinging
causes...quot;



                                                                                                                  40
Adhamjon’s SC notes                                                                            UZBEKISTAN

(B) is wrong because it is a sentence fragment: it has no independent clause.


(C) is wrong because it illogically states that the stinger results in the fact that the act results in the bee's
death.


(D) is wrong because, like (C), it says that the stinger results in the bee's death, and because, like (A), it
fails to use the possessive before the gerund.


What's wrong with (E)?


Itquot; does not refer to the possessive noun quot;honeybee's.quot; It refers to quot;stinger.quot; Nothing wrong with that. The
correct response is E.


Side note: GMAT writers do not subscribe to the rule that a pronoun cannot refer to a possessive noun.
There's a question in GMATPrep in which the correct answer has a pronoun that does just that.




A substance derived from the Madagascar periwinkle, which has proved useful in decreasing
mortality among young leukemia patients, is cultivated in China as part of a program to integrate
traditional herbal medicine into a contemporary system of health care.


(A) A substance derived from the Madagascar periwinkle, which has proved useful in decreasing
mortality among young leukemia patients,


(B) A derivative, which has proved useful in decreasing mortality among young leukemia patients,
of the Madagascar periwinkle,


(C) A Madagascar periwinkle derivative, which has proved useful in decreasing mortality among
young leukemia patients,


(D) The Madagascar periwinkle has a derivative which has proved useful in decreasing mortality
among young leukemia patients, that




                                                                                                                    41
Adhamjon’s SC notes                                                                          UZBEKISTAN

(E) The Madagascar periwinkle, a derivative of which has proved useful in decreasing mortality
among young leukemia patients,


EXPLANATION BY BOB:


I picked E.


I eliminated C for two reasons. First, it's the periwinkle, not the derivative, that is cultivated in China.
Second, C awkwardly and imprecisely strings three nouns together: quot;Madagascar periwinkle derivative.quot;
It sounds bad to this cultivated native speaker's ear, and it's not clear what the first noun is modifying. Is is
a derivative of the Madagascar periwinkle, or is it a Madagascar derivative of the periwinkle?




1. While depressed property values can hurt some large investors, they are potentially devastating
for home-owners, whose equity—in many cases representing a life’s savings—can plunge or even
disappear.
(A) they are potentially devastating for homeowners, whose
(B) they can potentially devastate homeowners in that their
(C) for homeowners they are potentially devastating, because their

(D) for homeowners, it is potentially devastating in that their(A)

(E) it can potentially devastate homeowners, whose


Why is (A) correct? the use of 'they' can either represent 'proporty values' or 'large investors'. Please let me
know.


EXPLANATION BY BOB:


Plural pronouns do not necessarily refer to the nearest plural noun, and singular pronouns do not
necessarily refer to the nearest singular noun. If that were true, there would be no such thing as ambiguous
pronoun reference. In the OG alone there are dozens of SC items in which the correct answer choice
includes a pronoun whose antecedent is not the nearest available noun. In addition to the item quoted
above (SC 159), here are just six more:


                                                                                                               42
Adhamjon’s SC notes                                                                         UZBEKISTAN

97. Iguanas have been an important food source in Latin America since prehistoric times, and they are
still prized as game animals by the campesinos....
The pronoun “they” refers to “iguanas,” not to the nearest plural noun “times.”


103. Students in the metropolitan school district are so lacking in math skills that it will be difficult to
absorb them into a city economy....
The pronoun “them” refers to “students,” not to the nearest plural noun “skills.”


152. ...a microbe never before seen on Earth that might escape from the laboratory and kill vast numbers
of humans who would have no natural defenses against it.
The pronoun “it” refers to “microbe,” not to the nearest singular noun “laboratory.”


153. A recording system was so secretly installed and operated in the Kennedy Oval Office that even
Theodore C. Sorensen, the White House counsel, did not know it existed.
The pronoun “it” refers to “system,” not to the nearest singular noun “Office.”


180. Quasars are so distant that their light has taken billions of years to reach the Earth; consequently,
we see them as they were during the formation of the universe.
The pronouns “them” and “they” refer to “quasars,” not to the nearest plural noun “years.”


251. The gyrfalcon, an Arctic bird of prey, has survived a close brush with extinction; its numbers are now
five times greater than when....
The pronoun “its” refers to “gyrfalcon,” not to the nearest singular noun “extinction.”


The rule is not so simply stated as quot;a pronoun will refer to the nearest available noun.quot; In fact, as all the
above examples show, if a pronoun agrees in number with the subject of the preceding clause, then that
subject will normally be the antecedent, even if there is another noun of the same number closer to the
pronoun.




quot;Jose's room is so messy that HIS mother calls HIM a pig.quot;


Per Manhattan SC, in the above sentence quot;HIMquot; is used incorrectly because it may not refer back to Jose.



                                                                                                                43
Adhamjon’s SC notes                                                                      UZBEKISTAN

What else could quot;HIMquot; refer to?


For the above sentence to be grammatically correct on GMAT the fix is -


quot;Jose's room is so messy that HIS mother calls JOSE a pig.quot;


Why is the objective pronoun incorrect [per GMAT terms]?


doesnt HIM indicate Jose to be the object?


Thnx.


Oh well! Never mind.


The subject here is Jose's room and not Jose, so we need to replace Him with Jose.


EXPLANATION BY BOB:


Some people believe it is inappropriate for a pronoun to refer to a noun in the possessive form (that is,
with an apostrophe s). I am certain, however, that GMAT writers do not subscribe to this rule -- I clearly
recall seeing a Sentence Correction item in GMATPrep in which the correct response has a pronoun that
refers to a possessive noun




Q: 800Bob, do you recall the other answer choices. Was there any option where the possessive noun was
not referred by a pronoun?


Yes. As I recall, the pronoun was in the non-underlined portion of the sentence and the antecedent was in
the underlined portion. Two or three answer choices had the antecedent without apostrophe+s, and the
other choices had the antecedent with apostrophe+s. The credited respose was one of the choices with the
possessive form. I remember it well, because it was the only question I answered incorrectly. Blind
allegiance to the quot;rulequot; that a pronoun may not refer to a possessive noun led me to choose an answer that
I should have known was unidiomatic.



                                                                                                            44
Adhamjon’s SC notes                                                                          UZBEKISTAN

I wish I had written the question down. Next time I come across it I will do so.


Here are three correct answers from 1000SC that violate this quot;rulequot;:


316. Frances Wright’s book on America contrasted the republicanism of the United States with what she
saw as the aristocratic and corrupt institutions of England. [Pronoun quot;shequot; refers to quot;Frances Wright'squot;]


458. Joplin’s faith in his opera “Tremonisha” was unshakable; in 1911 he published the score at his own
expense and decided to stage the work himself. [Pronoun quot;hequot; refers to quot;Joplin'squot;]


547. On stage, the force of Carrick’s personality and the vividness of his acting disguised the fact that he
was, as his surviving velvet suit shows, a short man. [Pronoun quot;hequot; refers to quot;Carrick'squot;]




1. In large doses, analgesics that work in the brain as antagonists to certain chemicals have caused
psychological disturbances in patients, which may limit their potential to relieve severe pain.
(A) which may limit their potential to relieve
(B) which may limit their potential for relieving
(C) which may limit such analgesics’ potential to relieve
(D) an effect that may limit their potential to relieve
(E) an effect that may limit the potential of such analgesics for relieving


I have a question on this problem, I was able to get till D & E and then picked D thinking that 'their' refers
to analgesics because as told by Bob, pronoun refers to subject of previous clause, which is quot;analgesicsquot; .
Also E looks to me lengthy
plz advise where i'm going wrong...thanx!!


EXPLANATION BY BOB:


I say E.


This is an interesting item. I'm not sure it's a real GMAT item, but I still think it's a useful one for
illustrating the dangers of following a rule of grammar too literally. Those of us with extremely analytical
minds would like to think that the rules of grammar can be expressed with precision. We must face the

                                                                                                            45
Adhamjon’s SC notes                                                                         UZBEKISTAN

fact, however, that language is not strictly logical.


I stand by the guideline that a pronoun will tend to refer to the subject of the preceding phrase if the
pronoun and subject agree in number. (We have seen many, many examples where the pronoun does not
automatically refer to the nearest noun.) So in this case it would seem that quot;theirquot; in choice D refers
grammatically and logically to quot;analgesics.quot; The problem with D, however, is that there are just too many
other plural nouns between quot;analgesicsquot; and quot;theirquot;: quot;antagonists,quot; quot;chemicals,quot; quot;disturbances,quot;
quot;patients.quot; For this reason it is better to write quot;of such analgesicsquot; than quot;their.quot;


Here's what I tell my students: If you have a choice that uses a pronoun and a choice repeats the noun,
generally you should go for the one that repeats the noun. A little repetition for the sake of clarity is not a
bad thing




Lincoln, discovering in young manhood the secret that the Yankee peddler has learned before him,
knew how to use a good story to generate good will.
(A) Lincoln, discovering in young manhood the secret that the Yankee peddler has learned before
him, knew
(B) Discovering in young manhood the secret that the Yankee peddler has learned before him,
Lincoln knew
(C) Lincoln, discovering the secret that the Yankee peddler had learned in young manhood before
him, knew
(D) In young manhood Lincoln discovered the secret that the Yankee peddler had learned before
him;
(E) Lincoln, discovered in young manhood the secret that the Yankee peddler had learned before
him, knew




EXPLANATION BY BOB:


(A) Lincoln, discovering in young manhood the secret that the Yankee peddler has learned before him,
knew
(B) Discovering in young manhood the secret that the Yankee peddler has learned before him, Lincoln
knew

                                                                                                              46
Adhamjon’s SC notes                                                                   UZBEKISTAN

Both wrong. Need past perfect quot;had learned.quot;


(C) Lincoln, discovering the secret that the Yankee peddler had learned in young manhood before him,
knew
Wrong. quot;In young manhoodquot; is misplaced.


(D) In young manhood Lincoln discovered the secret that the Yankee peddler had learned before him;
Wrong. Semicolon makes no sense here. A semicolon separates equivalent elements. Here there's an
independent clause before but a mere phrase after. If it were a colon, this sentence would be OK.


(E) Lincoln, discovered in young manhood the secret that the Yankee peddler had learned before him,
knew
Wrong. The phrase between commas begins with the past participle quot;discovered,quot; seeming to
suggest at first that Lincoln was discovered, but then making no sense at all when quot;the secretquot;
appears as the object of quot;discovered.quot;


So every choice is wrong. I believe that the answer is supposed to be D, but that somewhere along the way
into 1000SC a colon got changed into a semicolon.




Some psychiatric studies indicate that among distinguished artists the rates of manic depression and
major depression are ten to thirteen times as prevalent as in the population at large.
(A) the rates of manic depression and major depression are ten to thirteen times as prevalent as in
(B) the rates of manic depression and major depression are ten to thirteen times more
prevalent than in
(C) the rates of manic depression and major depression are ten to thirteen times more
prevalent when compared to
(D) manic depression and major depression are ten to thirteen times as prevalent when compared to
(E) manic depression and major depression are ten to thirteen times more prevalent than in


EXPLANATION BY BOB:




                                                                                                       47
Adhamjon’s SC notes                                                                     UZBEKISTAN

quot;rates...are more prevalentquot; is redundant and imprecise. Say either quot;the rates of manic depression and
major depression are greaterquot; or quot;manic depression and major depression are more prevalent.quot; It doesn't
really make sense to say that quot;ratesquot; are quot;more prevalent.quot;
Rate can never be fast/slow/prevalent...it is always greater or lesser...




The proposed health care bill would increase government regulation of health insurance, establish
standards that would gurantee wider access to people with past health problems and to workers
changing jobs who otherwise could be uncovered for months.


A) establish standards that would gurantee wider access to people with past health problems and to
workers changing jobs who


B) establishing standards that would gurantee wider access to people with past health problems and
to workers who are changing jobs and


C) to establish standards that would gurantee wider access to people with past health problems and
to workers who change jobs that


D) for establishing standards that would gurantee wider access for people with past health problems
and workers changing jobs who


E) for the establishment of standards that would gurantee wider access for people with past health
problems and workers who are changing jobs that


OA-B


EXPLANATION BY BOB:


In Question 8, A is impossible:


The proposed health care bill would increase government regulation of health insurance, establish
standards that would gurantee wider access to people with past health problems and to workers changing

                                                                                                          48
Adhamjon’s SC notes                                                                        UZBEKISTAN

jobs who
otherwise could be uncovered for months.


In this would-be sentence, we have quot;the... bill would increase..., establish...quot; You need quot;andquot; before
quot;establish.quot; You can't say quot;the bill would increase regulations, establish standards...quot; (unless there's
another verb later -- but there's not). It should be quot;the bill would increase regulations and establish
standards...quot;


What's wrong with B? quot;...wider access (1) to people with past health problems and (2) to workers who (a)
are changing jobs and (b) otherwise could be uncovered for months.quot;




It is an oversimplified view of cattle raising to say that all one has to do with cattle is leave them
alone while they feed themselves, corral them and to drive them to market when the time is ripe.


A) all one has to do with cattle is leave them alone while they feed themselves, coral them and to


B) all one has to do with cattle is to leave them alone to feed themselves, to corral them, and


C) all one has to do with cattle is leave them alone while they feed themselves and then corral them
and


D) the only thing that has to be done with cattle is leave them alone while they feed themselves,
corral them, and


E) the only thing that has to be done with cattle is to leave them alone while ttey feed themselves, to
corral them, and


EXPLANATION BY BOB:


In Question 11, A, B, and E are easily eliminated for lack of parallelism. They all attempt to present a list
of three verbs. To do so, you should repeat quot;toquot; always or never. That is, either:


quot;to leave..., to corral..., and to drive...quot;

                                                                                                           49
Adhamjon’s SC notes                                                                       UZBEKISTAN


or:


quot;to leave..., corral..., and drive...quot;


That leaves C and D.


D has a couple of problems. It says quot;the only thing that has to be done...quot; Then it lists three verbs. quot;Only
thingquot; implies one, not three. And verbs are not things. You can use an infinitive or a gerund as a noun,
but not the base form of the verb.




Oberlin College in Ohio was a renegade institution at its 1833 founding for deciding to accept both
men and women as students


A) at its 1833 founding for deciding to accept


B) for the decision at its 1833 founding to accept


C) when it was founded in 1833 for its decision to accept


D) in deciding at its founding in 1833 to accept


E) by deciding at its fouding in 1833 on the acceptance of


OA-D




Last year, land values in most parts of the pinelands rose almost so fast, and in some parts even
faster than what they did outside the pinelands.


A) so fast, and in some parts even faster than what they did


B) so fast, and in some parts even faster than, those

                                                                                                               50
Adhamjon’s SC notes                                                                         UZBEKISTAN


C) as fast, and in some parts even faster than, those


D) as fast as, and in some parts even faster than, those


E) as fast as, and in some parts even faster than what they did


EXPLANATION BY BOB:


D quot;What they didquot; is imprecise and unnecessary. You could say, for example, that quot;John ran as fast as, or
even faster than, Paulquot; or quot;John ran as fast as, or even faster than, Paul ranquot; or quot;John ran as fast as, or even
faster than, Paul did.quot; But you can't write quot;John ran as fast as, or even faster than, what Paul did.quot;




In the mid-1960’s a newly installed radar warning system mistook the rising of the moon as a
massive missile attack by the Soviets.


A) rising of the moon as a massive missile attack by the Soviets


B) rising of the moon for a massive Soviet missile attack


C) moon rising to a massive missile attack by the soviets


D) moon as it was rising for a massive Soviet missile attack


E) rise of the moon as a massive Soviet missile attack




EXPLANATION BY BOB:


B The correct idiom is quot;mistake x for y,quot; as in the book title quot;The Man Who Mistook His Wife for a Hat.quot;




                                                                                                             51
Adhamjon’s SC notes                                                                       UZBEKISTAN

With only 5 percent of the world’s population, United States citizens consume 28 percent of its
nonrenewable resources, drive more than one-third of its automobiles, and use 21 times more water
per capita than Europeans do.
A) With
B) As
C) Being
D) Despite having
E) Although accounting for


EXPLANATION BY BOB:


E The subject is quot;United States citizens.quot; You can't write quot;withquot; or quot;havingquot; because citizens don't have a
population. You can't write quot;asquot; or quot;beingquot; because they are not a number. The only choice that makes
sense is E: United States citizens quot;account forquot; 5 percent of the world's population.




Neanderthals had a vocal tract that resembled those of the apes and so were probably without
language, a shortcoming that may explain why they were supplanted by our own species.


A) Neanderthals had a vocal tract that resembled those of the apes


B) Neanderthals had a vocal tract resembling an ape’s


C) The vocal tracts of the Neanderthals resembled an ape’s


D) The Neanderthal’s vocal tracts resembled the apes’


E) The vocal tracts of the Neanderthals resembled those of the apes




EXPLANATION BY BOB:


B Singular quot;vocal tractquot; can be compared to quot;that of the apesquot; or quot;that of an apequot; or quot;an ape's,quot; but not to
plural quot;those of the apes.quot;

                                                                                                           52
Adhamjon’s SC notes                                                                        UZBEKISTAN


Q: B states that quot;Vocal Tract resembles an ape's quot;, so here vocal tract resembles to what part of ape's ? Is
this clear in this question . I dont know what i am asking might be very basic


quot;...had a vocal tract resembling an ape's...quot; means quot;...had a vocal tract resembling an ape's vocal tract.quot;
You can use the 's possessive to stand for the noun that's possessed, just as you can use possessive
pronouns such as quot;mine,quot; quot;yours,quot; and quot;hers.quot;


The baby has a face resembling his mother's face.
The baby has a face resembling his mother's.


The baby has a face resembling your face.
The baby has a face resembling yours.




Heavy commitment by an executive to a course of action, especially if it has worked well in the past,
makes it likely to miss signs of incipient trouble or misinterpret them when they do appear.


(A ) Heavy commitment by an executive to a course of action, especially if it has worked well in the
past, makes it likely to miss signs of incipient trouble or misinterpret them when they do appear.


(B ) An executive who is heavily committed to a course of action, especially one that worked well in
the past, makes missing signs of incipient trouble or misinterpreting ones likely when they do
appear.


(C ) An executive who is heavily committed to a course of action is likely to miss or misinterpret
signs of incipient trouble when they do appear, especially if it has worked well in the past.


(D ) Executives’ being heavily committed to a course of action, especially if it has worked well in the
past, makes them likely to miss signs of incipient trouble or misinterpreting them when they do
appear.


(E ) Being heavily committed to a course of action, especially one that has worked well in the past, is



                                                                                                              53
Adhamjon’s SC notes                                                                       UZBEKISTAN

likely to make an executive miss signs of incipient trouble or misinterpret them when they do
appear.


BOB:


Your quot;OAquot; is wrong. The correct response is E.




I am confused between A & E.
please somebody explain the usage of gerund & infinitive.
what difference it makes in this particular case.


According to some analysts, whatever its merits, the proposal to tax away all capital gains on short-
term investments would, if enacted, have a disastrous effect on Wall Street trading and employment.
(A) its merits, the proposal to tax
(B) its merits may be, the proposal of taxing
(C) its merits as a proposal, taxing
(D) the proposal’s merits, to tax
(E) the proposal’s merits are, taxing


EXPLANATION BY BOB:


The deciding difference between A and E is not infinitive versus gerund. What makes E wrong is that
quot;taxingquot; makes no sense as the subject. Later in the sentence we see quot;if enacted,quot; referring to the subject.
It makes no sense to speak of what would happen if taxing is enacted. The sentence is speculating on what
would happen if the proposal is enacted




One of Ronald Reagan’s first acts as President was to rescind President Carter’s directive that any
chemical banned on medical grounds in the United States be prohibited from sale to other countries.
(A) that any chemical banned on medical grounds in the United States be prohibited from sale to
other countries
(B) that any chemical be prohibited from sale to other countries that was banned on medical
                                                                                                           54
Adhamjons Sc Notes
Adhamjons Sc Notes
Adhamjons Sc Notes
Adhamjons Sc Notes
Adhamjons Sc Notes
Adhamjons Sc Notes
Adhamjons Sc Notes
Adhamjons Sc Notes
Adhamjons Sc Notes
Adhamjons Sc Notes
Adhamjons Sc Notes
Adhamjons Sc Notes
Adhamjons Sc Notes
Adhamjons Sc Notes
Adhamjons Sc Notes
Adhamjons Sc Notes
Adhamjons Sc Notes
Adhamjons Sc Notes
Adhamjons Sc Notes
Adhamjons Sc Notes
Adhamjons Sc Notes
Adhamjons Sc Notes
Adhamjons Sc Notes
Adhamjons Sc Notes
Adhamjons Sc Notes
Adhamjons Sc Notes
Adhamjons Sc Notes
Adhamjons Sc Notes
Adhamjons Sc Notes
Adhamjons Sc Notes
Adhamjons Sc Notes
Adhamjons Sc Notes
Adhamjons Sc Notes
Adhamjons Sc Notes
Adhamjons Sc Notes
Adhamjons Sc Notes
Adhamjons Sc Notes
Adhamjons Sc Notes
Adhamjons Sc Notes
Adhamjons Sc Notes
Adhamjons Sc Notes
Adhamjons Sc Notes
Adhamjons Sc Notes
Adhamjons Sc Notes
Adhamjons Sc Notes
Adhamjons Sc Notes
Adhamjons Sc Notes
Adhamjons Sc Notes
Adhamjons Sc Notes
Adhamjons Sc Notes
Adhamjons Sc Notes
Adhamjons Sc Notes
Adhamjons Sc Notes
Adhamjons Sc Notes
Adhamjons Sc Notes
Adhamjons Sc Notes
Adhamjons Sc Notes
Adhamjons Sc Notes
Adhamjons Sc Notes
Adhamjons Sc Notes
Adhamjons Sc Notes
Adhamjons Sc Notes
Adhamjons Sc Notes
Adhamjons Sc Notes
Adhamjons Sc Notes
Adhamjons Sc Notes
Adhamjons Sc Notes
Adhamjons Sc Notes
Adhamjons Sc Notes
Adhamjons Sc Notes
Adhamjons Sc Notes
Adhamjons Sc Notes
Adhamjons Sc Notes
Adhamjons Sc Notes
Adhamjons Sc Notes
Adhamjons Sc Notes
Adhamjons Sc Notes
Adhamjons Sc Notes
Adhamjons Sc Notes
Adhamjons Sc Notes
Adhamjons Sc Notes
Adhamjons Sc Notes
Adhamjons Sc Notes
Adhamjons Sc Notes
Adhamjons Sc Notes
Adhamjons Sc Notes
Adhamjons Sc Notes
Adhamjons Sc Notes
Adhamjons Sc Notes
Adhamjons Sc Notes
Adhamjons Sc Notes
Adhamjons Sc Notes
Adhamjons Sc Notes
Adhamjons Sc Notes
Adhamjons Sc Notes
Adhamjons Sc Notes
Adhamjons Sc Notes
Adhamjons Sc Notes
Adhamjons Sc Notes
Adhamjons Sc Notes

More Related Content

Similar to Adhamjons Sc Notes

Evolutionary Biology - Microevolution changing allelic frequencies
Evolutionary Biology - Microevolution changing allelic frequencies Evolutionary Biology - Microevolution changing allelic frequencies
Evolutionary Biology - Microevolution changing allelic frequencies Shaina Mavreen Villaroza
 
CAT 1994 Previous Year Question Paper with Answer Key
CAT 1994 Previous Year Question Paper with Answer KeyCAT 1994 Previous Year Question Paper with Answer Key
CAT 1994 Previous Year Question Paper with Answer KeyEneutron
 
Definition Essay Writing.pdf
Definition Essay Writing.pdfDefinition Essay Writing.pdf
Definition Essay Writing.pdfEllen Blackburn
 
Sentence correction(Parallelism based questions) .pptx
Sentence correction(Parallelism based questions) .pptxSentence correction(Parallelism based questions) .pptx
Sentence correction(Parallelism based questions) .pptxVigneshK635628
 
Entri65706850 Write Essays For College Students Cust
Entri65706850 Write Essays For College Students CustEntri65706850 Write Essays For College Students Cust
Entri65706850 Write Essays For College Students CustSandy Grasso
 
Cambridge Proficiency practice
Cambridge Proficiency practiceCambridge Proficiency practice
Cambridge Proficiency practiceSusan Bolling
 
Free Dedication Examples Of Thesis Papers 024 Free
Free Dedication Examples Of Thesis Papers  024 FreeFree Dedication Examples Of Thesis Papers  024 Free
Free Dedication Examples Of Thesis Papers 024 FreeBarb Tillich
 
Indian Culture And Heritage Essay
Indian Culture And Heritage EssayIndian Culture And Heritage Essay
Indian Culture And Heritage EssayAshley Bonham
 
How To Write A Abstract For A Research Paper
How To Write A Abstract For A Research PaperHow To Write A Abstract For A Research Paper
How To Write A Abstract For A Research PaperJulie Pate
 
Example Of Expository Essay Writing
Example Of Expository Essay WritingExample Of Expository Essay Writing
Example Of Expository Essay WritingLana Sorrels
 
014 Common Errors
014 Common Errors014 Common Errors
014 Common Errorsaptwano
 
College Essay 500 Word Limit 5 Simple Ways To Pa
College Essay 500 Word Limit 5 Simple Ways To PaCollege Essay 500 Word Limit 5 Simple Ways To Pa
College Essay 500 Word Limit 5 Simple Ways To PaJoy Smith
 
Does College Essay Need A Title. Online assignment writing service.
Does College Essay Need A Title. Online assignment writing service.Does College Essay Need A Title. Online assignment writing service.
Does College Essay Need A Title. Online assignment writing service.Jenny Gomez
 
Two Customizable ESL Writing Rubric Templates
Two Customizable ESL Writing Rubric TemplatesTwo Customizable ESL Writing Rubric Templates
Two Customizable ESL Writing Rubric TemplatesVeronica Rogers
 
Copper Sun Essay Questions
Copper Sun Essay QuestionsCopper Sun Essay Questions
Copper Sun Essay QuestionsLydia Jana
 

Similar to Adhamjons Sc Notes (20)

Evolutionary Biology - Microevolution changing allelic frequencies
Evolutionary Biology - Microevolution changing allelic frequencies Evolutionary Biology - Microevolution changing allelic frequencies
Evolutionary Biology - Microevolution changing allelic frequencies
 
CAT 1994 Previous Year Question Paper with Answer Key
CAT 1994 Previous Year Question Paper with Answer KeyCAT 1994 Previous Year Question Paper with Answer Key
CAT 1994 Previous Year Question Paper with Answer Key
 
Definition Essay Writing.pdf
Definition Essay Writing.pdfDefinition Essay Writing.pdf
Definition Essay Writing.pdf
 
Sentence correction(Parallelism based questions) .pptx
Sentence correction(Parallelism based questions) .pptxSentence correction(Parallelism based questions) .pptx
Sentence correction(Parallelism based questions) .pptx
 
COT2.pptx
COT2.pptxCOT2.pptx
COT2.pptx
 
Entri65706850 Write Essays For College Students Cust
Entri65706850 Write Essays For College Students CustEntri65706850 Write Essays For College Students Cust
Entri65706850 Write Essays For College Students Cust
 
Cambridge Proficiency practice
Cambridge Proficiency practiceCambridge Proficiency practice
Cambridge Proficiency practice
 
Free Dedication Examples Of Thesis Papers 024 Free
Free Dedication Examples Of Thesis Papers  024 FreeFree Dedication Examples Of Thesis Papers  024 Free
Free Dedication Examples Of Thesis Papers 024 Free
 
Florida State Essay
Florida State EssayFlorida State Essay
Florida State Essay
 
Indian Culture And Heritage Essay
Indian Culture And Heritage EssayIndian Culture And Heritage Essay
Indian Culture And Heritage Essay
 
How To Write A Abstract For A Research Paper
How To Write A Abstract For A Research PaperHow To Write A Abstract For A Research Paper
How To Write A Abstract For A Research Paper
 
Essays About Faith.pdf
Essays About Faith.pdfEssays About Faith.pdf
Essays About Faith.pdf
 
Unc College Essay
Unc College EssayUnc College Essay
Unc College Essay
 
Example Of Expository Essay Writing
Example Of Expository Essay WritingExample Of Expository Essay Writing
Example Of Expository Essay Writing
 
014 Common Errors
014 Common Errors014 Common Errors
014 Common Errors
 
College Essay 500 Word Limit 5 Simple Ways To Pa
College Essay 500 Word Limit 5 Simple Ways To PaCollege Essay 500 Word Limit 5 Simple Ways To Pa
College Essay 500 Word Limit 5 Simple Ways To Pa
 
English syntax
English syntaxEnglish syntax
English syntax
 
Does College Essay Need A Title. Online assignment writing service.
Does College Essay Need A Title. Online assignment writing service.Does College Essay Need A Title. Online assignment writing service.
Does College Essay Need A Title. Online assignment writing service.
 
Two Customizable ESL Writing Rubric Templates
Two Customizable ESL Writing Rubric TemplatesTwo Customizable ESL Writing Rubric Templates
Two Customizable ESL Writing Rubric Templates
 
Copper Sun Essay Questions
Copper Sun Essay QuestionsCopper Sun Essay Questions
Copper Sun Essay Questions
 

Recently uploaded

Call Girls Navi Mumbai Just Call 9907093804 Top Class Call Girl Service Avail...
Call Girls Navi Mumbai Just Call 9907093804 Top Class Call Girl Service Avail...Call Girls Navi Mumbai Just Call 9907093804 Top Class Call Girl Service Avail...
Call Girls Navi Mumbai Just Call 9907093804 Top Class Call Girl Service Avail...Dipal Arora
 
Call Girls in Mehrauli Delhi 💯Call Us 🔝8264348440🔝
Call Girls in Mehrauli Delhi 💯Call Us 🔝8264348440🔝Call Girls in Mehrauli Delhi 💯Call Us 🔝8264348440🔝
Call Girls in Mehrauli Delhi 💯Call Us 🔝8264348440🔝soniya singh
 
Call Girls In Sikandarpur Gurgaon ❤️8860477959_Russian 100% Genuine Escorts I...
Call Girls In Sikandarpur Gurgaon ❤️8860477959_Russian 100% Genuine Escorts I...Call Girls In Sikandarpur Gurgaon ❤️8860477959_Russian 100% Genuine Escorts I...
Call Girls In Sikandarpur Gurgaon ❤️8860477959_Russian 100% Genuine Escorts I...lizamodels9
 
Eni 2024 1Q Results - 24.04.24 business.
Eni 2024 1Q Results - 24.04.24 business.Eni 2024 1Q Results - 24.04.24 business.
Eni 2024 1Q Results - 24.04.24 business.Eni
 
M.C Lodges -- Guest House in Jhang.
M.C Lodges --  Guest House in Jhang.M.C Lodges --  Guest House in Jhang.
M.C Lodges -- Guest House in Jhang.Aaiza Hassan
 
Progress Report - Oracle Database Analyst Summit
Progress  Report - Oracle Database Analyst SummitProgress  Report - Oracle Database Analyst Summit
Progress Report - Oracle Database Analyst SummitHolger Mueller
 
Tech Startup Growth Hacking 101 - Basics on Growth Marketing
Tech Startup Growth Hacking 101  - Basics on Growth MarketingTech Startup Growth Hacking 101  - Basics on Growth Marketing
Tech Startup Growth Hacking 101 - Basics on Growth MarketingShawn Pang
 
VIP Call Girl Jamshedpur Aashi 8250192130 Independent Escort Service Jamshedpur
VIP Call Girl Jamshedpur Aashi 8250192130 Independent Escort Service JamshedpurVIP Call Girl Jamshedpur Aashi 8250192130 Independent Escort Service Jamshedpur
VIP Call Girl Jamshedpur Aashi 8250192130 Independent Escort Service JamshedpurSuhani Kapoor
 
Call Girls In Sikandarpur Gurgaon ❤️8860477959_Russian 100% Genuine Escorts I...
Call Girls In Sikandarpur Gurgaon ❤️8860477959_Russian 100% Genuine Escorts I...Call Girls In Sikandarpur Gurgaon ❤️8860477959_Russian 100% Genuine Escorts I...
Call Girls In Sikandarpur Gurgaon ❤️8860477959_Russian 100% Genuine Escorts I...lizamodels9
 
Call Girls In Radisson Blu Hotel New Delhi Paschim Vihar ❤️8860477959 Escorts...
Call Girls In Radisson Blu Hotel New Delhi Paschim Vihar ❤️8860477959 Escorts...Call Girls In Radisson Blu Hotel New Delhi Paschim Vihar ❤️8860477959 Escorts...
Call Girls In Radisson Blu Hotel New Delhi Paschim Vihar ❤️8860477959 Escorts...lizamodels9
 
Regression analysis: Simple Linear Regression Multiple Linear Regression
Regression analysis:  Simple Linear Regression Multiple Linear RegressionRegression analysis:  Simple Linear Regression Multiple Linear Regression
Regression analysis: Simple Linear Regression Multiple Linear RegressionRavindra Nath Shukla
 
Call Girls In Connaught Place Delhi ❤️88604**77959_Russian 100% Genuine Escor...
Call Girls In Connaught Place Delhi ❤️88604**77959_Russian 100% Genuine Escor...Call Girls In Connaught Place Delhi ❤️88604**77959_Russian 100% Genuine Escor...
Call Girls In Connaught Place Delhi ❤️88604**77959_Russian 100% Genuine Escor...lizamodels9
 
rishikeshgirls.in- Rishikesh call girl.pdf
rishikeshgirls.in- Rishikesh call girl.pdfrishikeshgirls.in- Rishikesh call girl.pdf
rishikeshgirls.in- Rishikesh call girl.pdfmuskan1121w
 
0183760ssssssssssssssssssssssssssss00101011 (27).pdf
0183760ssssssssssssssssssssssssssss00101011 (27).pdf0183760ssssssssssssssssssssssssssss00101011 (27).pdf
0183760ssssssssssssssssssssssssssss00101011 (27).pdfRenandantas16
 
Vip Dewas Call Girls #9907093804 Contact Number Escorts Service Dewas
Vip Dewas Call Girls #9907093804 Contact Number Escorts Service DewasVip Dewas Call Girls #9907093804 Contact Number Escorts Service Dewas
Vip Dewas Call Girls #9907093804 Contact Number Escorts Service Dewasmakika9823
 
Catalogue ONG NUOC PPR DE NHAT .pdf
Catalogue ONG NUOC PPR DE NHAT      .pdfCatalogue ONG NUOC PPR DE NHAT      .pdf
Catalogue ONG NUOC PPR DE NHAT .pdfOrient Homes
 
Keppel Ltd. 1Q 2024 Business Update Presentation Slides
Keppel Ltd. 1Q 2024 Business Update  Presentation SlidesKeppel Ltd. 1Q 2024 Business Update  Presentation Slides
Keppel Ltd. 1Q 2024 Business Update Presentation SlidesKeppelCorporation
 
Lowrate Call Girls In Laxmi Nagar Delhi ❤️8860477959 Escorts 100% Genuine Ser...
Lowrate Call Girls In Laxmi Nagar Delhi ❤️8860477959 Escorts 100% Genuine Ser...Lowrate Call Girls In Laxmi Nagar Delhi ❤️8860477959 Escorts 100% Genuine Ser...
Lowrate Call Girls In Laxmi Nagar Delhi ❤️8860477959 Escorts 100% Genuine Ser...lizamodels9
 

Recently uploaded (20)

Call Girls Navi Mumbai Just Call 9907093804 Top Class Call Girl Service Avail...
Call Girls Navi Mumbai Just Call 9907093804 Top Class Call Girl Service Avail...Call Girls Navi Mumbai Just Call 9907093804 Top Class Call Girl Service Avail...
Call Girls Navi Mumbai Just Call 9907093804 Top Class Call Girl Service Avail...
 
Call Girls in Mehrauli Delhi 💯Call Us 🔝8264348440🔝
Call Girls in Mehrauli Delhi 💯Call Us 🔝8264348440🔝Call Girls in Mehrauli Delhi 💯Call Us 🔝8264348440🔝
Call Girls in Mehrauli Delhi 💯Call Us 🔝8264348440🔝
 
Call Girls In Sikandarpur Gurgaon ❤️8860477959_Russian 100% Genuine Escorts I...
Call Girls In Sikandarpur Gurgaon ❤️8860477959_Russian 100% Genuine Escorts I...Call Girls In Sikandarpur Gurgaon ❤️8860477959_Russian 100% Genuine Escorts I...
Call Girls In Sikandarpur Gurgaon ❤️8860477959_Russian 100% Genuine Escorts I...
 
Eni 2024 1Q Results - 24.04.24 business.
Eni 2024 1Q Results - 24.04.24 business.Eni 2024 1Q Results - 24.04.24 business.
Eni 2024 1Q Results - 24.04.24 business.
 
M.C Lodges -- Guest House in Jhang.
M.C Lodges --  Guest House in Jhang.M.C Lodges --  Guest House in Jhang.
M.C Lodges -- Guest House in Jhang.
 
Progress Report - Oracle Database Analyst Summit
Progress  Report - Oracle Database Analyst SummitProgress  Report - Oracle Database Analyst Summit
Progress Report - Oracle Database Analyst Summit
 
Tech Startup Growth Hacking 101 - Basics on Growth Marketing
Tech Startup Growth Hacking 101  - Basics on Growth MarketingTech Startup Growth Hacking 101  - Basics on Growth Marketing
Tech Startup Growth Hacking 101 - Basics on Growth Marketing
 
VIP Call Girl Jamshedpur Aashi 8250192130 Independent Escort Service Jamshedpur
VIP Call Girl Jamshedpur Aashi 8250192130 Independent Escort Service JamshedpurVIP Call Girl Jamshedpur Aashi 8250192130 Independent Escort Service Jamshedpur
VIP Call Girl Jamshedpur Aashi 8250192130 Independent Escort Service Jamshedpur
 
Call Girls In Sikandarpur Gurgaon ❤️8860477959_Russian 100% Genuine Escorts I...
Call Girls In Sikandarpur Gurgaon ❤️8860477959_Russian 100% Genuine Escorts I...Call Girls In Sikandarpur Gurgaon ❤️8860477959_Russian 100% Genuine Escorts I...
Call Girls In Sikandarpur Gurgaon ❤️8860477959_Russian 100% Genuine Escorts I...
 
Best Practices for Implementing an External Recruiting Partnership
Best Practices for Implementing an External Recruiting PartnershipBest Practices for Implementing an External Recruiting Partnership
Best Practices for Implementing an External Recruiting Partnership
 
Call Girls In Radisson Blu Hotel New Delhi Paschim Vihar ❤️8860477959 Escorts...
Call Girls In Radisson Blu Hotel New Delhi Paschim Vihar ❤️8860477959 Escorts...Call Girls In Radisson Blu Hotel New Delhi Paschim Vihar ❤️8860477959 Escorts...
Call Girls In Radisson Blu Hotel New Delhi Paschim Vihar ❤️8860477959 Escorts...
 
Regression analysis: Simple Linear Regression Multiple Linear Regression
Regression analysis:  Simple Linear Regression Multiple Linear RegressionRegression analysis:  Simple Linear Regression Multiple Linear Regression
Regression analysis: Simple Linear Regression Multiple Linear Regression
 
Call Girls In Connaught Place Delhi ❤️88604**77959_Russian 100% Genuine Escor...
Call Girls In Connaught Place Delhi ❤️88604**77959_Russian 100% Genuine Escor...Call Girls In Connaught Place Delhi ❤️88604**77959_Russian 100% Genuine Escor...
Call Girls In Connaught Place Delhi ❤️88604**77959_Russian 100% Genuine Escor...
 
rishikeshgirls.in- Rishikesh call girl.pdf
rishikeshgirls.in- Rishikesh call girl.pdfrishikeshgirls.in- Rishikesh call girl.pdf
rishikeshgirls.in- Rishikesh call girl.pdf
 
0183760ssssssssssssssssssssssssssss00101011 (27).pdf
0183760ssssssssssssssssssssssssssss00101011 (27).pdf0183760ssssssssssssssssssssssssssss00101011 (27).pdf
0183760ssssssssssssssssssssssssssss00101011 (27).pdf
 
Vip Dewas Call Girls #9907093804 Contact Number Escorts Service Dewas
Vip Dewas Call Girls #9907093804 Contact Number Escorts Service DewasVip Dewas Call Girls #9907093804 Contact Number Escorts Service Dewas
Vip Dewas Call Girls #9907093804 Contact Number Escorts Service Dewas
 
KestrelPro Flyer Japan IT Week 2024 (English)
KestrelPro Flyer Japan IT Week 2024 (English)KestrelPro Flyer Japan IT Week 2024 (English)
KestrelPro Flyer Japan IT Week 2024 (English)
 
Catalogue ONG NUOC PPR DE NHAT .pdf
Catalogue ONG NUOC PPR DE NHAT      .pdfCatalogue ONG NUOC PPR DE NHAT      .pdf
Catalogue ONG NUOC PPR DE NHAT .pdf
 
Keppel Ltd. 1Q 2024 Business Update Presentation Slides
Keppel Ltd. 1Q 2024 Business Update  Presentation SlidesKeppel Ltd. 1Q 2024 Business Update  Presentation Slides
Keppel Ltd. 1Q 2024 Business Update Presentation Slides
 
Lowrate Call Girls In Laxmi Nagar Delhi ❤️8860477959 Escorts 100% Genuine Ser...
Lowrate Call Girls In Laxmi Nagar Delhi ❤️8860477959 Escorts 100% Genuine Ser...Lowrate Call Girls In Laxmi Nagar Delhi ❤️8860477959 Escorts 100% Genuine Ser...
Lowrate Call Girls In Laxmi Nagar Delhi ❤️8860477959 Escorts 100% Genuine Ser...
 

Adhamjons Sc Notes

  • 1. Adhamjon’s SC notes UZBEKISTAN Note: These are my SC notes collected from testmagic.com. The explanations are those of 800Bob, a Gmat instructor, who has got a perfect score. For new explanations, search on testmagic Gmat forum. The explanations are very informative and helpful. I learned a lot from them. I thank 800Bob for such awesome explanations. I hope my notes will help others. All the best. For the farmer who takes care to keep them cool, providing them with high-energy feed, and milking them regularly, Holstein cows are producing an average of 2,275 gallons of milk each per year. A. providing them with high-energy feed, and milking them regularly, Holstein cows are producing B. providing them with high-energy feed, and milked regularly, the Holstein cow produces C. provided with high-energy feed, and milking them regularly, Holstein cows are producing D. provided with high-energy feed, and milked regularly, the Holstein cow produces E. provided with high-energy feed, and milked regularly, Holstein cows will produce Bob’s EXPLANATION: B and D are easily eliminated because quot;themquot; does not agree with quot;cowquot;. A has a number of problems: 1. The modifying phrases quot;providing them with high-energy feedquot; and quot;milking them regularlyquot; are located between two clauses. It is not clear whether they modify the subject quot;whoquot; (= quot;farmersquot;) of the preceding clause or the subject quot;cowsquot; of the following clause. 2. The comma between the two modifying phrases should be omitted. 3. The use of the progressive tense quot;are producingquot; is inappropriate to describe a general scientific truth. C is wrong because quot;provided with high-energy feedquot; and quot;milking them regularlyquot; are not parallel and 1
  • 2. Adhamjon’s SC notes UZBEKISTAN because, like A, it uses the inappropriate progressive form quot;are producingquot;. E is perfect. The three adjectives quot;coolquot;, quot;providedquot;, and quot;milkedquot; are parallel, short for: quot;to keep them cool, keep them provided with high-energy feed, and keep them milked regularlyquot;. And the future form quot;will producequot; is appropriate for a general scientific truth. A mixture of poems and short fiction, Jean Toomer’s Cane has been called one of the three best novels ever written by Black Americans—the others being Richard Wright, author of Native Son, and Ralph Ellison, author of Invisible Man. A. Black Americans—the others being Richard Wright, author of Native Son, and Ralph Ellison, author of Invisible Man B. Black Americans—including Native Son by Richard Wright and Invisible Man by Ralph Ellison C. a Black American—including Richard Wright, author of Native Son, and Ralph Ellison, author of Invisible Man D. a Black American—the others being Richard Wright, author of Native Son, and Ralph Ellison, author of Invisible Man E. a Black American—the others being Richard Wright’s Native Son and Ralph Ellison’s Invisible Man Bob’s explanation: The best response is E. The only potential problem with E is quot;the three best novels ever written by a Black Americanquot;. But it is not wrong here to use the singular quot;Black Americanquot; because each of the three novels was written by just one person. Here are a couple of perfectly correct sentences that do something similar: 2
  • 3. Adhamjon’s SC notes UZBEKISTAN According to the American Dietetic Association, children who eat a healthy breakfast are more likely to have better concentration, problem-solving skills and eye-hand coordination. Those with a college degree had almost twice as much income as those with just a high school diploma. In the first example, each child eats just one breakfast. In the second, each person has just one college degree and just one high school diploma. Here are two correct sentences taken from Official Guides: Three out of four automobile owners in the United States also own a bicycle. (OG10, SC 252) Neanderthals had a vocal tract resembling an ape’s and so were probably without language, a shortcoming that may explain why they were supplanted by our own species. (OG Verbal, SC 59) For the first of these two examples there are a couple of answer choices that change quot;a bicyclequot; to quot;bicyclesquot;, but as the official explanation says, quot;the plural bicycles detracts from clarity by suggesting that multiple bicycles are owned by each person in question.quot; Similarly, for the second of these two examples there are three wrong answer choices that change quot;a vocal tractquot; to quot;vocal tractsquot;. So, in the example in question, choice B is wrong because it suggests that these novels each had multiple authors. It is also wrong because quot;includingquot; is imprecise, suggesting that Native Son and Invisible Man are Black Americans. You cannot eliminate E just because it includes the word quot;beingquot;. It is true that quot;beingquot; is often wrong or unnecessary, but to say that it is banned by the GMAT is ridiculous. Climatic shifts are so gradual as to be indistinguishable at first from ordinary fluctuations in the weather. (A) so gradual as to be indistinguishable (B) so gradual they can be indistinguishable 3
  • 4. Adhamjon’s SC notes UZBEKISTAN (C) so gradual that they are unable to be distinguished (D) gradual enough not to be distinguishable (E) gradual enough so that one cannot distinguish them Bob’s explanation: Use of the passive construction after able to or unable to is discouraged. It is not just awkward, but illogical. To be distinguished from ordinary fluctuations is not an ability that climatic shifts can possess or lack. To take a simpler example, look at this ugly sentence: This math problem is unable to be solved. Better: This math problem cannot be solved. In no other historical sighting did Halley 's comet cause such a worldwide sensation as did its return in 1910-1911. A did its return in 1910-1911 B had its 1910-1911 return C in its return of 1910-1911 D its return of 1910-1911 did E its return in 1910-1911 Bob’s explanation: If you begin the sentence this way, you can see more easily how to finish it: Halley's comet caused such a worldwide sensation in no other historical sighting as ... The comparison is between other historical sightings and that of 1910-1911, so you could finish the sentence this way: Halley's comet caused such a worldwide sensation in no other historical sighting as it did in its return of 1910-1911. Deleting the superfluous words quot;it didquot; you get: 4
  • 5. Adhamjon’s SC notes UZBEKISTAN Halley's comet caused such a worldwide sensation in no other historical sighting as in its return of 1910- 1911. So it is perfectly correct to write: In no other historical sighting did Halley's comet cause such a worldwide sensation as in its return of 1910-1911. The correct response is C. According to a recent study of consumer spending on prescription medications, increases in the sales of the 50 drugs that were advertised most heavily accounts for almost half of the $20.8 billion increase in drug spending last year, the remainder of which came from sales of the 9,850 prescription medicines that companies did not advertise or advertised very little. A. heavily accounts for almost half of the $20.8 billion increase in drug spending last year, the remainder of which came B. heavily were what accounted for almost half of the $20.8 billion increase in drug spending last year; the remainder of the increase coming C. heavily accounted for almost half of the $20.8 billion increase in drug spending last year, the remainder of the increase coming D. heavily, accounting for almost half of the $20.8 billion increase in drug spending last year, while the remainder of the increase came E. heavily, which accounted for almost half of the $20.8 billion increase in drug spending last year, with the remainder of it coming Bob’s explanation: I wish to respectfully disagree with a couple of the points made above. 1. quot;In GMAT we look for the best and not the right answer.quot; 5
  • 6. Adhamjon’s SC notes UZBEKISTAN The idea that sometimes the 5 choices are all bad and that we are to identify the least bad choice is a myth. There is never anything wrong with with correct response. The directions say to choose the quot;bestquot; answer, not the quot;correctquot; answer, because there can be more than one correct way to rephrase the sentence, but only one of the correct ways to do so will be found among the answer choices. 2. quot;B changes the meaning of the original sentence.quot; There is nothing in the directions to Sentence Correction about finding a choice that does not change the meaning of the original sentence. Indeed, sometimes it is necessary to change the meaning of the original, that is, when the original is illogical. Here, for example, is the orignal sentence of OG number 95: Unlike Schoenberg's 12-tone system that dominated the music of the postwar period, Bartók founded no school and left behind only a handful of disciples. In this form the sentence states that Schoenberg's 12-tone system was unlike Bartók. That's true, but it is obviously not what the writer intended. What matters is not the literal meaning of the original sentence, but the intended meaning. The objective is to find a choice that does not distort the intended meaning. The challenge sometimes is to determine from a poorly written original what the writer really intended. 3. quot;...there was a run on.quot; Choice C is not a run-on sentence. A run-on is two independent clauses treated as a single sentence. Here what follows the last comma is: the remainder of the increase coming from sales of the 9,850 prescription medicines that companies did not advertise or advertised That's not an independent clause. It could not stand alone: there's no conjugated verb. If you really want to know what grammatical form we have here... It's a nominative absolute. Check out Nominative absolute - Wikipedia, the free encyclopedia. There is nothing wrong with C, and so C is the best response. Over 75 percent of the energy produced in France derives from nuclear power, while in Germany it is just over 33 percent. 6
  • 7. Adhamjon’s SC notes UZBEKISTAN A. while in Germany it is just over 33 percent B. compared to Germany, which uses just over 33 percent C. whereas nuclear power accounts for just over 33 percent of the energy produced in Germany D. whereas just over 33 percent of the energy comes from nuclear power in Germany E. compared with the energy from nuclear power in Germany, where it is just over 33 percent Bob’s explanation: A is wrong because it refers illogically to the subject of the preceding clause 75 percent of the energy produced in France. It makes no sense to say that 75 percent of the energy produced in France is just over 33 percent in Germany. B is wrong because it compares 75 percent of the energy produced in France with Germany. D is wrong because it says that 33 percent of the energy produced in France comes from nuclear power in Germany. How can energy produced in France come from Germany? E is wrong because it is awkward and confusing, and specifically because the reference of the pronoun it is unclear. What is just over 33 percent? If it's nuclear power that's just over 33 percent, then just over 33 percent of what? The correct response is C Minivans carry as many as seven passengers and, compared with most sport utility vehicles, cost less, get better gas mileage, allow passengers to get in and out more easily, and have a smoother ride. A.Minivans carry as many as seven passengers and, compared with most sport utility vehicles, cost less, B.Minivans, which carry as many as seven passengers, compared with most sport utility vehicles, they cost less, C.Minivans carry as many as seven passengers, in comparison with most sport utility vehicles, and have a lower cost, they D.Minivans, carrying as many as seven passengers, compared with most sport utility vehicles, cost less, 7
  • 8. Adhamjon’s SC notes UZBEKISTAN E.Minivans, which carry as many as seven passengers, compared with most sport utility vehicles the cost is lower, and they Bob’s explanation: D is awkward and confusing in that it has two modifying phrases (quot;carrying as many as seven passengersquot; and quot;compared with most sport utility vehiclesquot;) between the subject quot;minivansquot; and the first verb quot;costquot;. Reading the sentence as choice D phrases it, when I get to quot;comparedquot; I first think it's the verb. Only when I get to quot;costquot; do I realize that the phrase beginning with quot;comparedquot; is not the predicate but a second modifier. D is confusing also in that it is not clear in what way minivans are being compared with most SUVs. Is the fact that minivans carry as many as seven passengers one of the differences? There is absolutely nothing wrong with choice A. It tells us two things about minivans: Minivans carry as many as seven passengers. Minivans (compared with most SUVs) cost less, etc. Since these two facts have the same subject, we can put them together, without repeating the subject, and inserting quot;andquot; between the verbs: Minivans carry as many as seven passengers and (compared with most SUVs) cost less, etc. It would be wrong to put a comma in front of quot;andquot;. On the other hand, the comma after quot;andquot; makes perfect sense: it's one of a pair of commas setting off the modifying phrase quot;compared with most SUVsquot;. The fact of some fraternal twins resembling each other greatly and others looking quite dissimilar highlights an interesting and often overlooked feature of fraternal-twin pairs, namely they vary considerably on a spectrum of genetic relatedness. A. The fact of some fraternal twins resembling each other greatly and others looking quite dissimilar highlights an interesting and often overlooked feature of fraternal-twin pairs, namely they vary considerably B. That some fraternal twins resemble each other greatly while others look quite dissimilar highlights an interesting and often overlooked feature of fraternal-twin pairs, namely that they vary 8
  • 9. Adhamjon’s SC notes UZBEKISTAN considerably C. With some fraternal twins resembling each other greatly and others looking quite dissimilar, it highlights an interesting and often overlooked feature of fraternal-twin pairs, namely considerable variation D. With some fraternal twins resembling each other greatly and others looking quite dissimilar, it is a fact that highlights an interesting and often overlooked feature of fraternal-twin pairs, namely a considerable variation E. Because some fraternal twins resemble each other greatly and others look quite dissimilar, this fact highlights an interesting and often overlooked feature of fraternal-twin pairs, namely they vary considerably EXPLANATION BY BOB: This is a case in which the subject is a whole clause: quot;that some fraternal twins resemble each other greatly while others look quite dissimilar.quot; That is the thing that is doing the highlighting. A IS WRONG: The fact of + noun + gerund...quot; - unidiomatic. quot;The fact that + noun + verb...quot; - better. quot;That + noun + verb....quot; - best. Example: quot;The fact of the GMAT being hard cannot be denied.quot; - unidiomatic and ugly. quot;The fact that the GMAT is hard cannot be denied.quot; - OK, but wordy. quot;That the GMAT is hard cannot be denied.quot; - best Malaria ravages more people than any disease, yet there are only a dozen laboratories in the world which are devoted to its study. (A) than any disease, yet there are only a dozen laboratories in the world which (B) than any disease, but there are only a dozen laboratories in the world which 9
  • 10. Adhamjon’s SC notes UZBEKISTAN (C) than does any disease, but only a dozen laboratories in the world (D) as any other disease, but only a dozen laboratories in the world (E) than any other disease, yet only a dozen laboratories in the world EXPLANATION BY BOB: in this case the word quot;otherquot; is enough to make it clear that the point of comparison is quot;malaria.quot; In correct choice E: quot;Malaria ravages more people than any other disease...quot; there can be no ambiguity. quot;Peoplequot; are not a disease. It would not be wrong to write: quot;Malaria ravages more people than does any other disease...quot; but it is wrong to omit quot;other.quot; Leaching, the recovery of copper from the drainage water of mines, as a method of the extraction of minerals, it was well established as early as the eighteenth century, but until about 25 years ago miners did not realize that bacteria take an active part in the process. A. as a method of the extraction of minerals, it was well established B. as a method of the extraction of minerals well established C. was a well-established method of mineral extraction D. was a well-established method of extracting mineral that was E. had been a method of mineral extraction, well established EXPLANATION BY BOB: There is no way the answer to number 1 can be A. It contains a double subject: quot;Leaching... it was...quot; In number 1, the past perfect is not necessary. The temporal relationship is made perfectly clear by the words quot;but until about 25 years ago.quot; Another example: quot;Until the 1960s New York was the most 10
  • 11. Adhamjon’s SC notes UZBEKISTAN populous state in the Union, but in the 1970s and ’80s the state began to experience an economic decline.quot; No need to write: quot;had been the most populous state.quot; In any case, the past perfect makes no sense the way it is used in E in that it suggests that leaching is no longer a method of mineral extraction. Past perfect would not be wrong, but it would have to be written something like: quot;had been a well-established method of mineral extraction.quot; The answer is C. According to United States census data, while there was about one-third of mothers with young children working outside the home in 1975, in 2000, almost two-thirds of those mothers were employed outside the home. A.while there was about one-third of mothers with young children working outside the home in 1975, in 2000, almost two-thirds of those mothers were employed outside the home. B.there were about one-third of mothers with young children who worked outside the home in 1975; in 2000, almost two-thirds of those mothers were employed outside the home in 1975 about one-third of mothers with young children worked outside the home; in 2000, almost two-thirds of such mothers were employed outside the home even though in 1975 there were about one-third of mothers with young children who worked outside the home, almost two-thirds of such mothers were employed outside the home in 2000 with about one-third of mothers with young children working outside the home in 1975, almost two-thirds of such mothers were employed outside the home in 2000 11
  • 12. Adhamjon’s SC notes UZBEKISTAN OA is C. 800BOB: a variety ofquot; quot;a number ofquot; quot;a total ofquot; ...plural quot;the variety ofquot; quot;the number ofquot; quot;the total ofquot; ...singular A variety of ideas were presented. The variety of ideas was mind-boggling. BOB: Here are three correct sentences from 1000 SC's in which quot;thatquot; refers to a noun earlier than the immediately preceding one: Out of America’s fascination with all things antique has grown a market for bygone styles of furniture and fixtures that is bringing back the chaise lounge, the overstuffed sofa, and the claw-footed bathtub. Dr. Sayre’s lecture recounted several little-known episodes in the relations between nations that illustrate what is wrong with alliances and treaties that do not have popular support. In good years, the patchwork of green fields that surrounds the San Joaquin Valley town bustles with farm workers, many of whom are in the area just for the season. 12
  • 13. Adhamjon’s SC notes UZBEKISTAN EXPLANATION BY BOB: Relative pronoun (that, who, which) agree with the immediately preceding noun. Subject and object pronouns (it, they, them) agree with the subject of the preceding clause or sentence I asked someone who is far far better than I am at determining the correct rules and grammar in general what his thoughts were on this subject and he (he's someone who had a perfect score on the exam) said, quot;In general, a participial phrase set off from the rest of the sentence with commas is understood to modify the subject of the sentencequot; It is misleading and oversimplistic to say that quot;whichquot; must always be preceded by a comma. The rule is that a nonrestrictive clause should be set off with commas, but a nonrestrictive clause does not always begin with quot;which,quot; and the presence of quot;whichquot; does not always signal a nonrestrictive clause. First of all, if the relative pronoun is the object of a preposition, whether the clause is restrictive or nonrestrictive, you cannot use quot;thatquot;; you must use quot;which.quot; Here are a couple of correct sentences from the OG that use quot;whichquot; after a preposition in a restrictive clause: El Nino, the periodic abnormal warming of the sea surface off Peru, is a phenomenon in which changes in the ocean and atmosphere combine to allow the warm water that has accumulated in the western Pacific to flow back to the east. As U.S. nuclear attack submarines prowl their familiar haunts deep within the oceans of the world these days, they increasingly are engaged in missions far different from the tasks for which they were built and for which their crews were trained over the last forty years Chicago, where industrial growth in the nineteenth century was more rapid than any other American city, was plauged by labour troubles like the Pullman Strikes of 1894. A) where industrial growth in the nineteenth century was more rapid than any other American city 13
  • 14. Adhamjon’s SC notes UZBEKISTAN B) which had industrial growth in the nineteenth century more rapid than that of other American cities C) which had growth industurally more rapid than any other American city in the ninteenth century D) whose industrial growth in the nineteenth century was more rapid than any other American city E) whose industrial growth in the nineteenth century was more rapid than of any other American city EXPLANATION BY BOB: Something's wrong here. All choices are wrong. Most other choices have comparison problems. A: says that Chicago's growth was quot;more rapid than any other American city.quot; Needs to be quot;more rapid than that of any other American city.quot; B: quot;had industrial growth... more rapid than that of other American cities.quot; Here quot;that ofquot; makes no sense. If you're going to start the comparison with quot;had,quot; you need to finish with a verb, as in quot;than other American cities had.quot; C: Adverb quot;industriallyquot; in quot;had growth industrially more rapidquot; seems to modify quot;more rapidquot; instead of quot;had growth.quot; D: like A, needs quot;that of.quot; E: Maybe this is supposed to be the correct answer and it's just mistyped. I see quot;ofquot; where I want to see quot;that of.quot; You can say either: which had industrial growth more rapid than other American cities had or: whose industrial growth was more rapid than that of other American cities But B confuses the two. quot;That of other American citiesquot; needs to be compared with some sort of possessive form, such as quot;the industrial growth of whichquot; or quot;whose industrial growth.quot; 14
  • 15. Adhamjon’s SC notes UZBEKISTAN Maybe an analogous but simpler example will help. You can say either: The area of Russia is greater than that of India. or: Russia has a greater area than India has. But it's imprecise to say: Russia has a greater area than that of India. BOB: before Australia was Australia, it was the antipodes MEANS It means: before the country that is now know as Australia got that name. This is not such a strange expression. Try googling on quot;before America was Americaquot; or quot;before England was Englandquot; and you'll get hundreds of hits. Although all the proceedings of the Communist party conference held in Moscow were not carried live, Soviet audiences have seen a great deal of coverage. (A) all the proceedings of the Communist party conference held in Moscow were not carried live (B) all the Communist party conference's Moscow proceedings were not carried live (C) all the Communist party conference Moscow proceedings have not been carried alive (D) not all the Communist party conference Moscow proceedings have been carried alive (E) not all the proceedings of the Communist party conference held in Moscow were carried live What is wrong with th given sentence? The OA is E 15
  • 16. Adhamjon’s SC notes UZBEKISTAN EXPLANATION BY BOB: A and E do not mean the same thing. A, which says that quot;all the proceedings were not carried live,quot; means that no proceeding was carried live. Though not grammatically incorrect, A is awkward and clearly not what the writer intended, since quot;audiences have seen a great deal of coverage.quot; There is a difference between quot;All X are not Yquot; and quot;Not all X are Y.quot; Consider the following: 1. All of us are not going. 2. Not all of us are going. In the first example, no one is going. In the second, some, but not all, are going. The construction used in A is a mistake commonly made by native speakers. Here are a few examples I found on the Internet: 1. quot;Althen is quick to acknowledge that all Americans are not white and middle-class.quot; Wrong. Should be: quot;...not all Americans are white and middle-class,quot; or quot;...Americans are not all white and middle-class.quot; 2. quot;All prime numbers are not odd.quot; Wrong. Should be: quot;Not all prime numbers are oddquot;, or quot;Prime numbers are not all odd.quot; 3. quot;All languages are not based on the English alphabet.quot; Wrong. Should be: quot;Not all languages are based on the English alphabet,quot; or quot;Languages are not all based on the English alphabet.quot; In a period of time when women typically have had a narrow range of choices, Mary Baker Eddy became a distinguished writer and the founder, architect, and builder of a growing church. 16
  • 17. Adhamjon’s SC notes UZBEKISTAN (A) In a period of time when women typically have (B) During a time in which typically women have (C) Typically, during a time when women (D) At a time when women typically (E) Typically in a time in which women EXPLANATION BY BOB: and B are wordy, but the real reason to eliminate them is quot;have.quot; The present perfect is wrong when describing an event that happened and was completed in the past. C and E are wrong because quot;typicallyquot; is misplaced. Quote: Originally Posted by zombie_in As per rules whenever we use quot;whichquot; it should have comma(,) in front of it Not true. You don't put a comma in front of quot;whichquot; when it follows a preposition. quot;In whichquot; is perfectly fine. California remains the state in which most Americans wish they could live. A historical novel is a novel in which the story is set among historical events. Would you really put a comma in front of which? California remains the state in, which most Americans wish they could live. (sic) A historical novel is a novel in, which the story is set among historical events. (sic Believed to originate from a small area on their foreheads, elephants emit low-frequency sounds that may be used as a secret language to communicate with other members of the herd. (A) Believed to originate from a small area on their foreheads, elephants emit low-frequency sounds that may be used 17
  • 18. Adhamjon’s SC notes UZBEKISTAN (B) Elephants emit low-frequency sounds that are believed to originate from a small area on their foreheads, and they may use this (C) Elephants emit low-frequency sounds, believed to originate from a small area on their foreheads, that they may use (D) Originating, it is believed, from a small area on their foreheads; elephants emit low-frequency sounds they may use (E) Originating, it is believed, from a small area on their foreheads, low-frequency sounds are emitted by elephants that may be used isnt 'they' in choice B refers to subject 'elephant' ........ EXPLANATION BY BOB: Problem with B is quot;this,quot; which is supposed to refer to quot;sounds.quot; Best response is C. Sunspots, vortices of gas associated with strong electro-magnetic activity, are visible as dark spots on the surface of the Sun but have never been sighted on the Sun's poles or equator. (A) are visible as dark spots on the surface of the Sun but have never been sighted on (B) are visible as dark spots that never have been sighted on the surface of the Sun (C) appear on the surface of the Sun as dark spots although never sighted at (D) appear as dark spots on the surface of the Sun, although never having been sighted at (E) appear as dark spots on the Suns surface, which have never been sighted on EXPLANATION: If you want quot;although never sightedquot; to modify quot;sunspots,quot; you would need to move it to the beginning of the sentence. Placed at the end, quot;althoughquot; needs to be followed by a subject and conjugated verb. quot;Atquot; versus quot;onquot; is a red herring -- they're both OK. It is counterproductive to disagree with the OG. The people who write the test say the answer is A. It's not 18
  • 19. Adhamjon’s SC notes UZBEKISTAN a typographical error. They offer a detailed explanation for why A is right and C is wrong. If one wants to get a high score on the GMAT, one needs to learn to think the way the test makers think. In the most common procedure for harvesting forage crops such as alfalfa, as much as 20 percent of the leaf and small-stem material, which is the most nutritious of all the parts of the plant, shattered and fell to the ground. (A) which is the most nutritious of all the parts of the plant, shattered and fell (B) the most nutritious of all parts of the plant, shatter and fall (C) the parts of the plant which were most nutritious, will shatter and fall (D) the most nutritious parts of the plant, shatters and falls (E) parts of the plant which are the most nutritious, have shattered and fallen OA D EXPLANATION BY BOB: There are two ways to look at the subject quot;20 percent of the leaf and small-stem material.quot; One way is to think it means 20 percent of two different things: quot;leafquot; and quot;small-stem material.quot; This way doesn't work because you can't say quot;20 percent of the leaf.quot; quot;Leafquot; is a countable noun. It would have to be quot;20 percent of the leaves.quot; The other way is to think it means 20 percent of the material. The material consists of leaves and small stems, and so it can be called quot;leaf and small-stem material.quot; This way works. So for me, the subject is singular. The present tense also works for me. So I would choose D. But I have a problem with D. If quot;leaf and small-stem materialquot; is singular, then I think it would be better for the following appositive to be singular also: quot;the most nutritious part of the plant.quot; All in all, not a good question. Clearly not GMAT-quality. 19
  • 20. Adhamjon’s SC notes UZBEKISTAN Pronoun 'they' 1. While depressed property values can hurt some large investors, they are potentially devastating for home-owners, whose equity—in many cases representing a life’s savings—can plunge or even disappear. (A) they are potentially devastating for homeowners, whose (B) they can potentially devastate homeowners in that their (C) for homeowners they are potentially devastating, because their (D) for homeowners, it is potentially devastating in that their(A) (E) it can potentially devastate homeowners, whose Why is (A) correct? the use of 'they' can either represent 'proporty values' or 'large investors'. Please let me know. EXPLANATION BY BOB: Plural pronouns do not necessarily refer to the nearest plural noun, and singular pronouns do not necessarily refer to the nearest singular noun. If that were true, there would be no such thing as ambiguous pronoun reference. In the OG alone there are dozens of SC items in which the correct answer choice includes a pronoun whose antecedent is not the nearest available noun. In addition to the item quoted above (SC 159), here are just six more: 97. Iguanas have been an important food source in Latin America since prehistoric times, and they are still prized as game animals by the campesinos.... The pronoun “they” refers to “iguanas,” not to the nearest plural noun “times.” 20
  • 21. Adhamjon’s SC notes UZBEKISTAN 103. Students in the metropolitan school district are so lacking in math skills that it will be difficult to absorb them into a city economy.... The pronoun “them” refers to “students,” not to the nearest plural noun “skills.” 152. ...a microbe never before seen on Earth that might escape from the laboratory and kill vast numbers of humans who would have no natural defenses against it. The pronoun “it” refers to “microbe,” not to the nearest singular noun “laboratory.” 153. A recording system was so secretly installed and operated in the Kennedy Oval Office that even Theodore C. Sorensen, the White House counsel, did not know it existed. The pronoun “it” refers to “system,” not to the nearest singular noun “Office.” 180. Quasars are so distant that their light has taken billions of years to reach the Earth; consequently, we see them as they were during the formation of the universe. The pronouns “them” and “they” refer to “quasars,” not to the nearest plural noun “years.” 251. The gyrfalcon, an Arctic bird of prey, has survived a close brush with extinction; its numbers are now five times greater than when.... The pronoun “its” refers to “gyrfalcon,” not to the nearest singular noun “extinction.” The rule is not so simply stated as quot;a pronoun will refer to the nearest available noun.quot; In fact, as all the above examples show, if a pronoun agrees in number with the subject of the preceding clause, then that subject will normally be the antecedent, even if there is another noun of the same number closer to the pronoun. According to GMAT which is correct This is me. or This is I. EXPLANATION BY BOB: I can't recall ever seeing either the word quot;mequot; or the word quot;Iquot; in a GMAT Sentence Correction item. 21
  • 22. Adhamjon’s SC notes UZBEKISTAN Strictly speaking, you should use the subject pronoun after forms of the verb quot;be.quot; It is I. It was they who arrived first. quot;Hello. May I speak to Bob?quot; quot;This is he.quot; If you were I... This is because quot;bequot; is not a transitive verb. A pronoun after quot;bequot; is a complement, not a direct or indirect object, and so grammatically speaking it is incorrect to use an object pronoun. But only grammar pedants like me speak this way. Normal people say, quot;It's me!quot; In any case, don't worry. This is never tested on the GMAT. In large doses, analgesics that work in the brain as antagonists to certain chemicals have caused psychological disturbances in patients, which may limit their potential to relieve severe pain. (A) which may limit their potential to relieve (B) which may limit their potential for relieving (C) which may limit such analgesics’ potential to relieve (D) an effect that may limit their potential to relieve (E) an effect that may limit the potential of such analgesics for relieving I have a question on this problem, I was able to get till D & E and then picked D thinking that 'their' refers to analgesics because as told by Bob, pronoun refers to subject of previous clause, which is quot;analgesicsquot; . Also E looks to me lengthy plz advise where i'm going wrong...thanx!! EXPLANATION BY BOB: I say E. 22
  • 23. Adhamjon’s SC notes UZBEKISTAN This is an interesting item. I'm not sure it's a real GMAT item, but I still think it's a useful one for illustrating the dangers of following a rule of grammar too literally. Those of us with extremely analytical minds would like to think that the rules of grammar can be expressed with precision. We must face the fact, however, that language is not strictly logical. I stand by the guideline that a pronoun will tend to refer to the subject of the preceding phrase if the pronoun and subject agree in number. (We have seen many, many examples where the pronoun does not automatically refer to the nearest noun.) So in this case it would seem that quot;theirquot; in choice D refers grammatically and logically to quot;analgesics.quot; The problem with D, however, is that there are just too many other plural nouns between quot;analgesicsquot; and quot;theirquot;: quot;antagonists,quot; quot;chemicals,quot; quot;disturbances,quot; quot;patients.quot; For this reason it is better to write quot;of such analgesicsquot; than quot;their.quot; Here's what I tell my students: If you have a choice that uses a pronoun and a choice repeats the noun, generally you should go for the one that repeats the noun. A little repetition for the sake of clarity is not a bad thing. Classical guitar was neither prestigious nor was often played in concert halls until it was revived by Andres Segovia in the mid-twentieth century, having been won over by the instrument's sound despite its relative obscurity. A. Classical guitar was neither prestigious nor was often played in concert halls until it was revived by Andres Segovia in the mid-twentieth century, having been won over by the instrument's sound despite its relative obscurity. B. Classical guitar was neither prestigious nor played often in concert halls until it was revived by Andres Segovia in the mid-twentieth century, having been won over by the instrument's sound despite its relative obscurity. C. Classical guitar was not prestigious and was not often played in concert halls until Andres Segovia revived it in the mid-twentieth century, after he was won over by the sound despite the instrument's relative obscurity. D. Classical guitar did not have prestige nor was it performed often in concert halls until its revival by Andres Segovia, who in the mid-twentieth century was won over by the instrument's sound despite its relative obscurity. 23
  • 24. Adhamjon’s SC notes UZBEKISTAN E. Classical guitar was neither prestigious nor was often played in concert halls until Andres Segovia revived it in the mid-twentieth century, when he was won over by the sound of the relatively obscure instrument. OA C EXPLANATION BY BOB: For me the best choice is C. A and E are wrong because quot;neither prestigious nor was often playedquot; is not parallel. A and B are wrong because the participial phrase quot;having been won...quot; seems to modify (illogically) the subject of the sentence, quot;classical guitar.quot; D is wrong because quot;playedquot; is changed to quot;performed.quot; You don't perform an instrument; you play it or perform on it. D also has an ambiguous quot;its.quot; I see nothing wrong with C. With its plan to develop seven and a half acres of shore land, Cleveland is but one of a large number of communities on the Great Lakes that is looking to its waterfront as a way to improve the quality of urban life and attract new businesses. (A) is looking to its waterfront as a way to improve the quality of urban life and attract (B) is looking at its waterfront to improve the quality of urban life and attract (C) are looking to their waterfronts to improve the quality of urban life and attract (D) are looking to its waterfront as a way of improving the quality of urban life and attracting 24
  • 25. Adhamjon’s SC notes UZBEKISTAN (E) are looking at their waterfronts as a way they can improve the quality of urban life and attract OA C EXPLANATION BY BOB: The verb must be quot;arequot;: Cleveland is one of many communities that are... quot;Thatquot; refers to quot;communitiesquot; and is therefore plural. Note the difference between quot;one of the [plural noun] isquot; and quot;one of the [plural noun] that arequot;: One of the most powerful driving forces behind recycling is the threat of legislation that would require companies to take more responsibility for the disposal of their products. Twenty-two feet long and 10 feet in diametier, the AM-1 is one of the many new satellites that are part of a 15-year effort to subject the interactions of Earth's atmosphere, oceans, and land surfaces to detailed scrutiny from space. Last edited by 800Bob : 07-08-2005 at 06:20 PM. Looking at: Physically act of seeing sthg/someone Looking to: Depending on, turning to, expecting sthg from.. Look to the waterfront to improve my business.. I have started considering that waterways are a good means to improve business.. Look up to you - Turn to you for advice.. Are they both acceptable? Mr. King agrees it is possible that planets orbiting .... could ... Mr. King agrees that it is possible for planets orbiting ... to .... Any ideas? 25
  • 26. Adhamjon’s SC notes UZBEKISTAN EXPLANATION BY BOB: quot;It is possible that planets couldquot; is redundant. It's better not to use quot;could,quot; quot;might,quot; or quot;mayquot; with quot;possiblequot; or quot;possibly.quot; Bob, is it okay to say It is possible for him to do something I am not sure about this ''for x to do'' usage. What do u say? Yes, it is possible for you to say that. A 1972 agreement between Canada and the United States reduced the amount of phosphates that municipalities had been allowed to dump into the Great Lakes. (A) reduced the amount of phosphates that municipalities had been allowed to dump (B) reduced the phosphate amount that municipalities had been dumping (C) reduces the phosphate amount municipalities have been allowed to dump (D) reduced the amount of phosphates that municipalities are allowed to dump (E) reduces the amount of phosphates allowed for dumping by municipalities EXPLANATION BY BOB: agree with D. An agreement cannot reduce the amount that had been allowed. That previous amount cannot be changed. There is an allowable amount. That amount was bigger before the agreement, and it is smaller now as a result of the agreement. Analogy: Let's say the legislature has changed the speed limit. Which sounds better? 26
  • 27. Adhamjon’s SC notes UZBEKISTAN 1) The legislature increased the maximum speed that one had been allowed to drive. 2) The legislature increased the maximum speed that one is allowed to drive. I'll go with 2. Past perfect is used to suggest that something is over and done with before the main action of the past. It would be OK to say: The 1972 agreement replaced the amount of phosphates that had been allowed with a new amount. But if you are talking about changing an amount, then the amount existed before and continues to exist after the 1972 agreement. The present tense is used to refer to something that exists for all time. Copernicus revealed that the Earth and the planets all revolve around the Sun. quot;Revolvequot; is in the present tense because it is an action that was then and continues to take place. It would be wrong to say: quot;...revealed that the Earth and the planets revolved...quot; or quot;had revolved.quot; In the sentence under dicussion, there was and continues to be an amount of phosphates that municipalities can dump. The 1972 agreement reduced the amount, but the amount continues to exist. I'll try one more example. Suppose I started a new diet last week. I formerly ate all the red meat I wanted. Under the new diet I allow myself to eat only 100 grams a day. The diet I started last week reduced the amount of red meat I am allowed to eat to 100 grams a day. It would be wrong to say: The diet I started last week reduced the amount of red meat I had been allowed to eat to 100 grams a day. GOOD INSIGHT How can a agreement reduce something that has already been dumped? Choice D is perfect. A is ,initially, awfully tempting. Brilliant Question!! 27
  • 28. Adhamjon’s SC notes UZBEKISTAN 17. Bob Wilber became Sidney Bechet’s student and protege when he was nineteen and, for a few years in the 1940’s, came as close to being a carbon copy of the jazz virtuoso in performance as anyone has ever come. (A) as anyone has ever come (B) as anyone ever had been (C) as anyone ever had done (D) that anyone ever did (E) that anyone ever OA-C EXPLANATION BY BOB: The quot;OAquot; is wrong. The best answer is A. B and C are both wrong because quot;everquot; is misplaced. Adverbs of frequency (quot;always,quot; quot;never,quot; quot;often,quot; quot;seldom,quot; quot;ever,quot; etc.) must be placed between the auxiliary verb and the past participle. Should be: quot;as anyone had ever done.quot; There is nothing wrong with using the present perfect quot;has comequot; in A. It means that Wilber came as close to being a carbon copy of the jazz virtuoso as anyone had come before or has come since. Consider the following examples: quot;In 1960 Johnson scored as many goals as anyone had ever scored in one year.quot; quot;In 1960 Johnson scored as many goals as anyone has ever scored in one year.quot; 28
  • 29. Adhamjon’s SC notes UZBEKISTAN In the first example, Johnson's number of goals in 1960 is compared only with previous years. In the second example his number of goals in 1960 is compared with all years before and since. Either past perfect or present perfect would be correct. But C puts quot;everquot; in the wrong place and so cannot be correct. I see nothing wrong with A. For a few years in the 1940s Wilber came very close to being a carbon copy of his teacher. To this day, no one has ever come closer. I have seen quite a few cases in which the so-called quot;OAquot; is wrong. 1. A survey by the National Council of Churches showed that in 1986 there were 20,376 female ministers, almost 9 percent of the nation's clergy, twice as much as 1977. (A) twice as much as 1977 (B) twice asn many as 1977 (C) double what it was in 1977 (D) double the figure for 1977 (E) a number double that of 1977's SPOILER: OA - D 2. According to surveys by the National Institute on Drug Abuse, about 20 percent of young adults used cocaine in 1979, doubling those reported in the 1977 survey. (A) doubling those reported in the 1977 survey (B) to double the number the 1977 survey reported (C) twice those the 1977 survey reported (D) twice as much as those reported in the 1977 survey (E) twice the number reported in the 1977 survey 29
  • 30. Adhamjon’s SC notes UZBEKISTAN SPOILER: OA - E EXPLANATION BY BOB: Yes, the dictionary says that quot;twicequot; is an adverb, but it's perfectly idiomatic to say quot;twice the number.quot; Putting quot;twicequot; before quot;the numberquot; does not make quot;twicequot; an adjective. If it were an adjective you'd have to place it after quot;the,quot; but you can't say quot;the twice number.quot; VERY GOOD POINT HERE: In April 1997, Hillary Rodham Clinton hosted an all-day White House scientific conference on new findings that indicates a child’s acquiring language, thinking, and emotional skills as an active process that may be largely completed before age three. A. that indicates a child’s acquiring language, thinking, and emotional skills as B. that are indicative of a child acquiring language, thinking, and emotional skills as C. to indicate that when a child acquires language, thinking, and emotional skills, that it is D. indicating that a child’s acquisition of language, thinking, and emotional skills is E. indicative of a child’s acquisition of language, thinking, and emotional skills as GOOD EXPLANATION BY BOB: I don't like E at all. quot;Indicative ofquot; is idiomatically correct in some cases, but not here. quot;Indicative ofquot; should be followed by a noun. Example: Some believe that facial characteristics are indicative of personality traits. The noun quot;personality traitsquot; is what is indicated. Example: Are these averages indicative of what a typical family pays for medical care? The noun phrase quot;what a typical family paysquot; is what is indicated. 30
  • 31. Adhamjon’s SC notes UZBEKISTAN But to say: quot;indicative of a child's acquisition of ... skills as an active processquot; is awkward and imprecise. It's not the skills that are indicated; it's the fact that these skills are an active process. I see nothing wrong with D. The objection some have raised is that the participial phrase quot;indicating that a child's acquisition...quot; might could be felt to modify the subject quot;Hillary Rodham Clintonquot; instead of the immediately preceding noun quot;findings.quot; For a participial phrase at the end of a sentence to modify a distant subject instead of an adjacent noun, it must be separated from the rest of the sentence with a comma, like this: Hillary Rodham Clinton hosted an all-day White House scientific conference on new findings, indicating that a child’s acquisition of language, thinking, and emotional skills is an active process that may be largely completed before age three. Without a comma between quot;findingsquot; and quot;indicating,quot; choice D is not ambiguous The health commissioner said that the government had implemented strict measures to eradicate the contaminated food and, despite the recent illnesses, it will try to prevent the outbreak from reccurring in the future. A) it will try B) that it tried C) it had tried D) it would try E) that it would try OA-E EXPLANATION : 31
  • 32. Adhamjon’s SC notes UZBEKISTAN quot;Wouldquot; is used to refer to the future from the point of view of the past. quot;He said he would arrive shortly.quot; The Health commissioner said X and Y ... X == that the government had implemented strict measures to eradicate the contaminated food Y= that the government would try .. I hope this will make u clear smudge ... A good Question . Testing Parallelism .. I was also wrong initiallly , Now i learned it. Thanks for the question. Employment costs rose 2.8% in the 12months that ended in sept,slightly(less than they did) in the year that ended in the previous quarter. A...... B.lower than C.lower than they were D.lower than they did E.less than OA-A EXPLANATION BY BOB: The sentence is not comparing employment costs of two different periods. It is comparing how much the costs rose in two different periods. Employment costs rose by a smaller amount in the 12 months that ended in September than they rose in the 12 months that ended in June. Employment costs rose 2.8% in the more recent period, slightly less than they rose in the earlier period. 32
  • 33. Adhamjon’s SC notes UZBEKISTAN There seem to be two different versions of this item in GMATPrep. One is the version imperfectly typed at the top of this thread. That version actually looks like this: In addition to her work on the Miocene hominid fossil, Mary Leakey contributed to archaeology through her discovery of the earliest direct evidence of hominid activity and through her painstaking documentation of East African cave paintings. A) Mary Leakey contributed to archaeology through her discovery of the earliest direct evidence of hominid activity and through her painstaking documentation of B) Mary Leakey contributed to archaeology by her discovery of the earliest direct evidence of hominid activity and painstakingly documenting C) Mary Leakey was a contributor to archaeology by discovering the earliest direct evidence of hominid activity and with her painstaking documentation of D) Mary Leakey's contributions to archaeology include her discovery of the earliest direct evidence of hominid activity and painstakingly documenting E) Mary Leakey's contributions to archaeology include her discovering the earliest direct evidence of hominid activity and painstaking documentation of Correct response: SPOILER: A The other version looks like this: In addition to her work on the Miocene hominid fossil record, Mary Leakey contributed to archaeology with her discovery of the earliest direct evidence of hominid activity and painstakingly documenting East African cave paintings. A) Leakey contributed to archaeology with her discovery of the earliest direct evidence of hominid 33
  • 34. Adhamjon’s SC notes UZBEKISTAN activity and painstakingly documenting B) Leakey contributed to archaeology by her discovery of the earliest direct evidence of hominid activity and by painstakingly documenting C) Leakey was a contributor to archaeology with her discovery of the earliest direct evidence of hominid activvity and with her painstaking documentation of D) Leakey's contributions to archaeology include her discovery of the earliest direct evidence of hominid activity and her painstaking documentation of E) Leakey's contributions to archaeology include discovering the earliest direct evidence of hominid activity and painstaking documentation of Correct response: SPOILER: D Dr. Sayre’s lecture recounted several little-known episodes in the relations between nations that illustrates what is wrong with alliances and treaties that do not have popular support. (A) relations between nations that illustrates (B) relation of one nation with another that illustrates (C) relations between nations that illustrate (D) relation of one nation with another and illustrate (E) relations of nations that illustrates EXPLANATION: The correct response is C. Subject-verb agreement problem in A, B, and E. Should be: quot;...several little-known episodes ... that illustrate...quot; quot;Betweenquot; is just fine here. From The Columbia Guide to Standard American English: 34
  • 35. Adhamjon’s SC notes UZBEKISTAN quot;It is often argued that between should be used to express a relationship involving two of something, and among should express relationships involving three or more, but in fact that generalization does not describe the way English has long used these prepositions. Between can be used of as many items as you like if the relationship is one-to-one, however much it may be repeated with different partners: Economic relations between Great Britain, France, and Italy [or between some members of the EEC] are tense at present.quot; The computer software being designed for a project studying native American access to higher education not only meets the needs of that study, but also has the versatility and power of facilitiating similar research endeavors. a. but also has the versatility and power of facilititating b. but also have the power to facilitate c. but it also has the versatility and power to facilitate d. and also have the versatility and power of facilititating e. it also has such versatility and power that it can facilitate The answer is SPOILER: B but I think it should be SPOILER: C EXPLANATION BY BOB: The correctly typed sentence reads like this: The computer software being designed for a project studying Native American access to higher education will not only meet the needs of that study, but also have the versatility and power to facilitate similar research endeavors. It doesn't matter whether the subject is singular or plural. After will you must write have, never has. 35
  • 36. Adhamjon’s SC notes UZBEKISTAN Q; C is still correct then, not just not idiomatically? No. The question as typed at the top of this thread has no correct answer. quot;Not only meetsquot; and quot;but it also hasquot; are not parallel. It should be quot;not only meetsquot; and quot;but also has.quot; A is close, but quot;power of facilitatingquot; is not idiomatic. Should be quot;power to facilitate.quot; Not only did the systematic clearing of forests in the United States create farmland (especially in the Northeast) and gave consumers relatively inexpensive houses and furniture, but it also caused erosion and very quickly deforested whole regions. 1. Not only did the systematic clearing of forests in the United States create farmland (especially in the Northeast) and gave consumers relatively inexpensive houses and furniture, but it also 2. Not only did the systematic clearing of forests in the United States create farmland (especially in the Northeast), which gave consumers relatively inexpensive houses and furniture, but also 3. The systematic clearing of forests in the United States, creating farmland (especially in the Northeast) and giving consumers relatively inexpensive houses and furniture, but also 4. The systematic clearing of forests in the United States created farmland (especially in the Northeast) and gave consumers relatively inexpensive houses and furniture, but it also 5. The systematic clearing of forests in the United States not only created farmland (especially in the Northeast), giving consumers relatively inexpensive houses and furniture, but it EXPLANATION BY BOB: B is no good. quot;Not only did the systematic clearing... create...quot; doesn't parallel quot;but also gave...quot; Furthermore, B erroneously uses quot;whichquot; to refer to the whole preceding clause. The correct response is D. ANOTHER EXPLANATION BY BOB: I think the best choice is D. The first clause says that the systematic clearing did some good things (created farmland and provided inexpensive houses and furniture); the second clause says that it also did 36
  • 37. Adhamjon’s SC notes UZBEKISTAN some bad things (caused erosion and deforested whole regions). quot;Not alsoquot; generally calls for a later quot;but also,quot; but quot;but alsoquot; does not necessarily call for an earlier quot;not only.quot; Example: Rio de Janeiro has some of the world's most famous beaches, but it also has some of the world's most notorious slums New hardy varieties of rice show promise of producing high yields without the costly requirements of irrigation and application of commercial fertilizer by earlier high-yielding varieties. (A) requirements of irrigation and application of commercial fertilizer by earlier high-yielding varieties (B) requirements by earlier high-yielding varieties of application of commercial fertilizer and irrigation (C) requirements for application of commercial fertilizer and irrigation of earlier high-yielding varieties (D) application of commercial fertilizer and irrigation that was required by earlier high-yielding varieties (E) irrigation and application of commercial fertilizer that were required by earlier high-yielding varieties EXPLANATION BYBOB: The word order in C is really bad. quot;Costlyquot; is placed before and therefore modifies quot;requirementsquot; instead of quot;irrigation and application of commercial fertilizer.quot; Does quot;application of commercial fertilizer and irrigationquot; mean quot;(1) application of commercial fertilizer and (2) irrigationquot; or does it mean quot;application of (1) commercial fertilizer and (2) irrigationquot;? And then there's the placement of the phrase quot;of earlier high-yielding varieties.quot; Does only quot;irrigationquot; relate to the earlier varieties, or do both quot;applicationquot; and quot;irrigationquot; relate to them? 37
  • 38. Adhamjon’s SC notes UZBEKISTAN What's wrong with E? Passive? So what? It's a persistent myth that passive is bad. Sometimes passive is not only acceptable, but necessary. Look for example at OG11 SC questions 67 and 86 By the time peace and happiness will have come to the planet,many lives will be wasted. A...................... B. come to the planet, many lives will have been wasted C. will have come to the planet, many lives will have been wasted D. shall have come to the planet, many lives shall be wasted E. would have come to the planet, many lives would have been wasted EXPLANATION BY BOB: I agree with the OA. In the first clause we need the present tense. With quot;by the time,quot; as with quot;when,quot; quot;before,quot; quot;while,quot; quot;after,quot; and quot;until,quot; we use the present with a future meeting. By the time we get there, the store will be closed. When I see you tomorrow, I will give you the dollar I owe you. He will turn off the lights before he leaves. Will you take care of the plants while I am on vacation? We won't go until we are finished. So already we see that the correct response must be B. In the second clause the future perfect quot;will have beenquot; is correct. The future perfect refers to an event that happens in the past from the point of view of the future. The time when peace and happiness come is in the future. Lives will be wasted before that time. 38
  • 39. Adhamjon’s SC notes UZBEKISTAN One pervasive theory explains the introduction of breakfast cereals in the early 1900s as a result of the growing number of automobiles, which led to a decline in horse ownership and a subsequent grain glut; by persuading people to eat what had previously been horse feed, market equilibrium was restored. (A) by persuading people to eat what had previously been horse feed, market equilibrium was restored (B) persuading people to eat what had previously been horse feed restored market equilibrium (C) by persuading people to eat what had previously been horse feed, it restored market equilibrium (D) the persuasion of people to eat what had previously been horse feed restored market equilibrium (E) market equilibrium was restored when people were persuaded to eat former horse feed EXPLANATION BY BOB: The correct response is indeed B. A is wrong because the phrase quot;by persuading...quot; illogically modifies quot;market equilibrium.quot; C is wrong because quot;itquot; has no logical antecedent. D is wrong because quot;persuasion of peoplequot; is ambiguous. Are the people persuading or are they being persuaded? E is wrong for a couple of reasons: 1) The word quot;whenquot; suggests that quot;market equilibrium was restoredquot; and quot;people were persuadedquot; merely happened concurrently. The writer's intent is to say that quot;people were persuadedquot; was the cause of quot;market equilibrium was restored.quot; 2) quot;Former horse feedquot; is awkward and imprecise. Sounds like feed for former horses. 39
  • 40. Adhamjon’s SC notes UZBEKISTAN As the honeybee's stinger is heavily barbed, staying where it is inserted, this results in the act of stinging causing the bee to sustain a fatal injury. A.As the honeybee's stinger is heavily barbed, staying where it is inserted, this results in the act of stinging causing B.As the heavily barbed singer of the honey bee stays where it is inserted, with the result that the act of stinging causes C.The honeybee's stinger, heavily barbed and staying where it is inserted, results in the fact that the act of stinging causes D.The heavily barbed stinger of the honeybee stays where it is inserted, and results in the act of stinging causing E.The honeybee's stinger is heavily barbed and stays where it is inserted, with the result that the act of stinging causes EXPLANATION BY BOB: I do not agree with (C). It is not the bee's stinger that causes the bee's death. It is the fact that the stinger is barbed and stays where it is inserted. For me, the best answer is (E). Seems like a quot;GMAT qualityquot; question to me. It has four absolutely wrong answer choices and one answer choice with absolutely nothing wrong. (A) is wrong because it uses quot;thisquot; to refer not to a specific noun but to the whole idea expressed earlier in the sentence. (A) is also wrong because of quot;results in the act of stinging causing...quot; quot;Causingquot; is a gerund and needs to be preceded by the possessive, which would be the very ugly quot;the act of stinging's causing...quot; The only way around that would be to write something like quot;results in the fact that the act of stinging causes...quot; 40
  • 41. Adhamjon’s SC notes UZBEKISTAN (B) is wrong because it is a sentence fragment: it has no independent clause. (C) is wrong because it illogically states that the stinger results in the fact that the act results in the bee's death. (D) is wrong because, like (C), it says that the stinger results in the bee's death, and because, like (A), it fails to use the possessive before the gerund. What's wrong with (E)? Itquot; does not refer to the possessive noun quot;honeybee's.quot; It refers to quot;stinger.quot; Nothing wrong with that. The correct response is E. Side note: GMAT writers do not subscribe to the rule that a pronoun cannot refer to a possessive noun. There's a question in GMATPrep in which the correct answer has a pronoun that does just that. A substance derived from the Madagascar periwinkle, which has proved useful in decreasing mortality among young leukemia patients, is cultivated in China as part of a program to integrate traditional herbal medicine into a contemporary system of health care. (A) A substance derived from the Madagascar periwinkle, which has proved useful in decreasing mortality among young leukemia patients, (B) A derivative, which has proved useful in decreasing mortality among young leukemia patients, of the Madagascar periwinkle, (C) A Madagascar periwinkle derivative, which has proved useful in decreasing mortality among young leukemia patients, (D) The Madagascar periwinkle has a derivative which has proved useful in decreasing mortality among young leukemia patients, that 41
  • 42. Adhamjon’s SC notes UZBEKISTAN (E) The Madagascar periwinkle, a derivative of which has proved useful in decreasing mortality among young leukemia patients, EXPLANATION BY BOB: I picked E. I eliminated C for two reasons. First, it's the periwinkle, not the derivative, that is cultivated in China. Second, C awkwardly and imprecisely strings three nouns together: quot;Madagascar periwinkle derivative.quot; It sounds bad to this cultivated native speaker's ear, and it's not clear what the first noun is modifying. Is is a derivative of the Madagascar periwinkle, or is it a Madagascar derivative of the periwinkle? 1. While depressed property values can hurt some large investors, they are potentially devastating for home-owners, whose equity—in many cases representing a life’s savings—can plunge or even disappear. (A) they are potentially devastating for homeowners, whose (B) they can potentially devastate homeowners in that their (C) for homeowners they are potentially devastating, because their (D) for homeowners, it is potentially devastating in that their(A) (E) it can potentially devastate homeowners, whose Why is (A) correct? the use of 'they' can either represent 'proporty values' or 'large investors'. Please let me know. EXPLANATION BY BOB: Plural pronouns do not necessarily refer to the nearest plural noun, and singular pronouns do not necessarily refer to the nearest singular noun. If that were true, there would be no such thing as ambiguous pronoun reference. In the OG alone there are dozens of SC items in which the correct answer choice includes a pronoun whose antecedent is not the nearest available noun. In addition to the item quoted above (SC 159), here are just six more: 42
  • 43. Adhamjon’s SC notes UZBEKISTAN 97. Iguanas have been an important food source in Latin America since prehistoric times, and they are still prized as game animals by the campesinos.... The pronoun “they” refers to “iguanas,” not to the nearest plural noun “times.” 103. Students in the metropolitan school district are so lacking in math skills that it will be difficult to absorb them into a city economy.... The pronoun “them” refers to “students,” not to the nearest plural noun “skills.” 152. ...a microbe never before seen on Earth that might escape from the laboratory and kill vast numbers of humans who would have no natural defenses against it. The pronoun “it” refers to “microbe,” not to the nearest singular noun “laboratory.” 153. A recording system was so secretly installed and operated in the Kennedy Oval Office that even Theodore C. Sorensen, the White House counsel, did not know it existed. The pronoun “it” refers to “system,” not to the nearest singular noun “Office.” 180. Quasars are so distant that their light has taken billions of years to reach the Earth; consequently, we see them as they were during the formation of the universe. The pronouns “them” and “they” refer to “quasars,” not to the nearest plural noun “years.” 251. The gyrfalcon, an Arctic bird of prey, has survived a close brush with extinction; its numbers are now five times greater than when.... The pronoun “its” refers to “gyrfalcon,” not to the nearest singular noun “extinction.” The rule is not so simply stated as quot;a pronoun will refer to the nearest available noun.quot; In fact, as all the above examples show, if a pronoun agrees in number with the subject of the preceding clause, then that subject will normally be the antecedent, even if there is another noun of the same number closer to the pronoun. quot;Jose's room is so messy that HIS mother calls HIM a pig.quot; Per Manhattan SC, in the above sentence quot;HIMquot; is used incorrectly because it may not refer back to Jose. 43
  • 44. Adhamjon’s SC notes UZBEKISTAN What else could quot;HIMquot; refer to? For the above sentence to be grammatically correct on GMAT the fix is - quot;Jose's room is so messy that HIS mother calls JOSE a pig.quot; Why is the objective pronoun incorrect [per GMAT terms]? doesnt HIM indicate Jose to be the object? Thnx. Oh well! Never mind. The subject here is Jose's room and not Jose, so we need to replace Him with Jose. EXPLANATION BY BOB: Some people believe it is inappropriate for a pronoun to refer to a noun in the possessive form (that is, with an apostrophe s). I am certain, however, that GMAT writers do not subscribe to this rule -- I clearly recall seeing a Sentence Correction item in GMATPrep in which the correct response has a pronoun that refers to a possessive noun Q: 800Bob, do you recall the other answer choices. Was there any option where the possessive noun was not referred by a pronoun? Yes. As I recall, the pronoun was in the non-underlined portion of the sentence and the antecedent was in the underlined portion. Two or three answer choices had the antecedent without apostrophe+s, and the other choices had the antecedent with apostrophe+s. The credited respose was one of the choices with the possessive form. I remember it well, because it was the only question I answered incorrectly. Blind allegiance to the quot;rulequot; that a pronoun may not refer to a possessive noun led me to choose an answer that I should have known was unidiomatic. 44
  • 45. Adhamjon’s SC notes UZBEKISTAN I wish I had written the question down. Next time I come across it I will do so. Here are three correct answers from 1000SC that violate this quot;rulequot;: 316. Frances Wright’s book on America contrasted the republicanism of the United States with what she saw as the aristocratic and corrupt institutions of England. [Pronoun quot;shequot; refers to quot;Frances Wright'squot;] 458. Joplin’s faith in his opera “Tremonisha” was unshakable; in 1911 he published the score at his own expense and decided to stage the work himself. [Pronoun quot;hequot; refers to quot;Joplin'squot;] 547. On stage, the force of Carrick’s personality and the vividness of his acting disguised the fact that he was, as his surviving velvet suit shows, a short man. [Pronoun quot;hequot; refers to quot;Carrick'squot;] 1. In large doses, analgesics that work in the brain as antagonists to certain chemicals have caused psychological disturbances in patients, which may limit their potential to relieve severe pain. (A) which may limit their potential to relieve (B) which may limit their potential for relieving (C) which may limit such analgesics’ potential to relieve (D) an effect that may limit their potential to relieve (E) an effect that may limit the potential of such analgesics for relieving I have a question on this problem, I was able to get till D & E and then picked D thinking that 'their' refers to analgesics because as told by Bob, pronoun refers to subject of previous clause, which is quot;analgesicsquot; . Also E looks to me lengthy plz advise where i'm going wrong...thanx!! EXPLANATION BY BOB: I say E. This is an interesting item. I'm not sure it's a real GMAT item, but I still think it's a useful one for illustrating the dangers of following a rule of grammar too literally. Those of us with extremely analytical minds would like to think that the rules of grammar can be expressed with precision. We must face the 45
  • 46. Adhamjon’s SC notes UZBEKISTAN fact, however, that language is not strictly logical. I stand by the guideline that a pronoun will tend to refer to the subject of the preceding phrase if the pronoun and subject agree in number. (We have seen many, many examples where the pronoun does not automatically refer to the nearest noun.) So in this case it would seem that quot;theirquot; in choice D refers grammatically and logically to quot;analgesics.quot; The problem with D, however, is that there are just too many other plural nouns between quot;analgesicsquot; and quot;theirquot;: quot;antagonists,quot; quot;chemicals,quot; quot;disturbances,quot; quot;patients.quot; For this reason it is better to write quot;of such analgesicsquot; than quot;their.quot; Here's what I tell my students: If you have a choice that uses a pronoun and a choice repeats the noun, generally you should go for the one that repeats the noun. A little repetition for the sake of clarity is not a bad thing Lincoln, discovering in young manhood the secret that the Yankee peddler has learned before him, knew how to use a good story to generate good will. (A) Lincoln, discovering in young manhood the secret that the Yankee peddler has learned before him, knew (B) Discovering in young manhood the secret that the Yankee peddler has learned before him, Lincoln knew (C) Lincoln, discovering the secret that the Yankee peddler had learned in young manhood before him, knew (D) In young manhood Lincoln discovered the secret that the Yankee peddler had learned before him; (E) Lincoln, discovered in young manhood the secret that the Yankee peddler had learned before him, knew EXPLANATION BY BOB: (A) Lincoln, discovering in young manhood the secret that the Yankee peddler has learned before him, knew (B) Discovering in young manhood the secret that the Yankee peddler has learned before him, Lincoln knew 46
  • 47. Adhamjon’s SC notes UZBEKISTAN Both wrong. Need past perfect quot;had learned.quot; (C) Lincoln, discovering the secret that the Yankee peddler had learned in young manhood before him, knew Wrong. quot;In young manhoodquot; is misplaced. (D) In young manhood Lincoln discovered the secret that the Yankee peddler had learned before him; Wrong. Semicolon makes no sense here. A semicolon separates equivalent elements. Here there's an independent clause before but a mere phrase after. If it were a colon, this sentence would be OK. (E) Lincoln, discovered in young manhood the secret that the Yankee peddler had learned before him, knew Wrong. The phrase between commas begins with the past participle quot;discovered,quot; seeming to suggest at first that Lincoln was discovered, but then making no sense at all when quot;the secretquot; appears as the object of quot;discovered.quot; So every choice is wrong. I believe that the answer is supposed to be D, but that somewhere along the way into 1000SC a colon got changed into a semicolon. Some psychiatric studies indicate that among distinguished artists the rates of manic depression and major depression are ten to thirteen times as prevalent as in the population at large. (A) the rates of manic depression and major depression are ten to thirteen times as prevalent as in (B) the rates of manic depression and major depression are ten to thirteen times more prevalent than in (C) the rates of manic depression and major depression are ten to thirteen times more prevalent when compared to (D) manic depression and major depression are ten to thirteen times as prevalent when compared to (E) manic depression and major depression are ten to thirteen times more prevalent than in EXPLANATION BY BOB: 47
  • 48. Adhamjon’s SC notes UZBEKISTAN quot;rates...are more prevalentquot; is redundant and imprecise. Say either quot;the rates of manic depression and major depression are greaterquot; or quot;manic depression and major depression are more prevalent.quot; It doesn't really make sense to say that quot;ratesquot; are quot;more prevalent.quot; Rate can never be fast/slow/prevalent...it is always greater or lesser... The proposed health care bill would increase government regulation of health insurance, establish standards that would gurantee wider access to people with past health problems and to workers changing jobs who otherwise could be uncovered for months. A) establish standards that would gurantee wider access to people with past health problems and to workers changing jobs who B) establishing standards that would gurantee wider access to people with past health problems and to workers who are changing jobs and C) to establish standards that would gurantee wider access to people with past health problems and to workers who change jobs that D) for establishing standards that would gurantee wider access for people with past health problems and workers changing jobs who E) for the establishment of standards that would gurantee wider access for people with past health problems and workers who are changing jobs that OA-B EXPLANATION BY BOB: In Question 8, A is impossible: The proposed health care bill would increase government regulation of health insurance, establish standards that would gurantee wider access to people with past health problems and to workers changing 48
  • 49. Adhamjon’s SC notes UZBEKISTAN jobs who otherwise could be uncovered for months. In this would-be sentence, we have quot;the... bill would increase..., establish...quot; You need quot;andquot; before quot;establish.quot; You can't say quot;the bill would increase regulations, establish standards...quot; (unless there's another verb later -- but there's not). It should be quot;the bill would increase regulations and establish standards...quot; What's wrong with B? quot;...wider access (1) to people with past health problems and (2) to workers who (a) are changing jobs and (b) otherwise could be uncovered for months.quot; It is an oversimplified view of cattle raising to say that all one has to do with cattle is leave them alone while they feed themselves, corral them and to drive them to market when the time is ripe. A) all one has to do with cattle is leave them alone while they feed themselves, coral them and to B) all one has to do with cattle is to leave them alone to feed themselves, to corral them, and C) all one has to do with cattle is leave them alone while they feed themselves and then corral them and D) the only thing that has to be done with cattle is leave them alone while they feed themselves, corral them, and E) the only thing that has to be done with cattle is to leave them alone while ttey feed themselves, to corral them, and EXPLANATION BY BOB: In Question 11, A, B, and E are easily eliminated for lack of parallelism. They all attempt to present a list of three verbs. To do so, you should repeat quot;toquot; always or never. That is, either: quot;to leave..., to corral..., and to drive...quot; 49
  • 50. Adhamjon’s SC notes UZBEKISTAN or: quot;to leave..., corral..., and drive...quot; That leaves C and D. D has a couple of problems. It says quot;the only thing that has to be done...quot; Then it lists three verbs. quot;Only thingquot; implies one, not three. And verbs are not things. You can use an infinitive or a gerund as a noun, but not the base form of the verb. Oberlin College in Ohio was a renegade institution at its 1833 founding for deciding to accept both men and women as students A) at its 1833 founding for deciding to accept B) for the decision at its 1833 founding to accept C) when it was founded in 1833 for its decision to accept D) in deciding at its founding in 1833 to accept E) by deciding at its fouding in 1833 on the acceptance of OA-D Last year, land values in most parts of the pinelands rose almost so fast, and in some parts even faster than what they did outside the pinelands. A) so fast, and in some parts even faster than what they did B) so fast, and in some parts even faster than, those 50
  • 51. Adhamjon’s SC notes UZBEKISTAN C) as fast, and in some parts even faster than, those D) as fast as, and in some parts even faster than, those E) as fast as, and in some parts even faster than what they did EXPLANATION BY BOB: D quot;What they didquot; is imprecise and unnecessary. You could say, for example, that quot;John ran as fast as, or even faster than, Paulquot; or quot;John ran as fast as, or even faster than, Paul ranquot; or quot;John ran as fast as, or even faster than, Paul did.quot; But you can't write quot;John ran as fast as, or even faster than, what Paul did.quot; In the mid-1960’s a newly installed radar warning system mistook the rising of the moon as a massive missile attack by the Soviets. A) rising of the moon as a massive missile attack by the Soviets B) rising of the moon for a massive Soviet missile attack C) moon rising to a massive missile attack by the soviets D) moon as it was rising for a massive Soviet missile attack E) rise of the moon as a massive Soviet missile attack EXPLANATION BY BOB: B The correct idiom is quot;mistake x for y,quot; as in the book title quot;The Man Who Mistook His Wife for a Hat.quot; 51
  • 52. Adhamjon’s SC notes UZBEKISTAN With only 5 percent of the world’s population, United States citizens consume 28 percent of its nonrenewable resources, drive more than one-third of its automobiles, and use 21 times more water per capita than Europeans do. A) With B) As C) Being D) Despite having E) Although accounting for EXPLANATION BY BOB: E The subject is quot;United States citizens.quot; You can't write quot;withquot; or quot;havingquot; because citizens don't have a population. You can't write quot;asquot; or quot;beingquot; because they are not a number. The only choice that makes sense is E: United States citizens quot;account forquot; 5 percent of the world's population. Neanderthals had a vocal tract that resembled those of the apes and so were probably without language, a shortcoming that may explain why they were supplanted by our own species. A) Neanderthals had a vocal tract that resembled those of the apes B) Neanderthals had a vocal tract resembling an ape’s C) The vocal tracts of the Neanderthals resembled an ape’s D) The Neanderthal’s vocal tracts resembled the apes’ E) The vocal tracts of the Neanderthals resembled those of the apes EXPLANATION BY BOB: B Singular quot;vocal tractquot; can be compared to quot;that of the apesquot; or quot;that of an apequot; or quot;an ape's,quot; but not to plural quot;those of the apes.quot; 52
  • 53. Adhamjon’s SC notes UZBEKISTAN Q: B states that quot;Vocal Tract resembles an ape's quot;, so here vocal tract resembles to what part of ape's ? Is this clear in this question . I dont know what i am asking might be very basic quot;...had a vocal tract resembling an ape's...quot; means quot;...had a vocal tract resembling an ape's vocal tract.quot; You can use the 's possessive to stand for the noun that's possessed, just as you can use possessive pronouns such as quot;mine,quot; quot;yours,quot; and quot;hers.quot; The baby has a face resembling his mother's face. The baby has a face resembling his mother's. The baby has a face resembling your face. The baby has a face resembling yours. Heavy commitment by an executive to a course of action, especially if it has worked well in the past, makes it likely to miss signs of incipient trouble or misinterpret them when they do appear. (A ) Heavy commitment by an executive to a course of action, especially if it has worked well in the past, makes it likely to miss signs of incipient trouble or misinterpret them when they do appear. (B ) An executive who is heavily committed to a course of action, especially one that worked well in the past, makes missing signs of incipient trouble or misinterpreting ones likely when they do appear. (C ) An executive who is heavily committed to a course of action is likely to miss or misinterpret signs of incipient trouble when they do appear, especially if it has worked well in the past. (D ) Executives’ being heavily committed to a course of action, especially if it has worked well in the past, makes them likely to miss signs of incipient trouble or misinterpreting them when they do appear. (E ) Being heavily committed to a course of action, especially one that has worked well in the past, is 53
  • 54. Adhamjon’s SC notes UZBEKISTAN likely to make an executive miss signs of incipient trouble or misinterpret them when they do appear. BOB: Your quot;OAquot; is wrong. The correct response is E. I am confused between A & E. please somebody explain the usage of gerund & infinitive. what difference it makes in this particular case. According to some analysts, whatever its merits, the proposal to tax away all capital gains on short- term investments would, if enacted, have a disastrous effect on Wall Street trading and employment. (A) its merits, the proposal to tax (B) its merits may be, the proposal of taxing (C) its merits as a proposal, taxing (D) the proposal’s merits, to tax (E) the proposal’s merits are, taxing EXPLANATION BY BOB: The deciding difference between A and E is not infinitive versus gerund. What makes E wrong is that quot;taxingquot; makes no sense as the subject. Later in the sentence we see quot;if enacted,quot; referring to the subject. It makes no sense to speak of what would happen if taxing is enacted. The sentence is speculating on what would happen if the proposal is enacted One of Ronald Reagan’s first acts as President was to rescind President Carter’s directive that any chemical banned on medical grounds in the United States be prohibited from sale to other countries. (A) that any chemical banned on medical grounds in the United States be prohibited from sale to other countries (B) that any chemical be prohibited from sale to other countries that was banned on medical 54